529 CHAPTER TWENTY-SEVEN Mucopolysaccharidoses The mucopolysaccharidoses are a group of heritable lysosomal enzyme disorders (often autosomal recessive) that lead to accumulation of glycosaminoglycans (GAGs) (see also Chapter 29, Metabolic medicine). The clinical presentations are variable, with a spectrum of multisystem features including MSK (joint/ finger contractures, trigger fingers, carpal tunnel syndrome, kyphoscoliosis), upper airways obstruction (snoring, sleep apnoea from large tongue, tonsils and adenoids, recurrent infections and glue ear/deafness), learning difficulties, recurrent hernias, characteristic appearance and cardiac valve problems and cardiomyopathy. The diagnosis is made by demonstration of raised urinary GAGs, with confirmation by either genetic or white cell enzyme testing. Mitochondrial disorders Mitochondrial disorders arise from mutations in either the mitochondrial DNA (maternal inheritance pattern) or the genes within the nucleus that are involved in transportation of proteins into the mitochondria. Mitochondrial disorders result in abnormalities in the adenosine triphosphate (ATP) energy production pathways. In addition to neurological and cardiovascular problems, mitochondrial disorders affect the MSK system, predominately causing hypotonia (see Chapter 9, Genetics, and Chapter 29, Metabolic medicine, for further details). Genetic susceptibility in autoimmune conditions There are also a number of genetic susceptibility genes, which play a role in the development of autoimmune conditions, many of which have abnormalities affecting the MSK system (such as inflammatory arthritis). There is an increased incidence of most autoimmune conditions when another family member is affected and there is a higher concordance in monozygotic compared to dizygotic twins. The human leukocyte antigen (HLA) system is linked with a number of autoimmune conditions. The HLA system allows our immune system to distinguish the self-made proteins from proteins made by foreign invaders (such as viruses and bacteria). There are many different normal variations within each HLA gene facilitating each person’s immune system ability to react to a wide range of foreign proteins. For example, a range of normal variations of HLA genes exists which affect the risk of developing JIA; HLA-B27 is strongly associated with enthesitis-related arthritis. Table 27.2 Examples of single gene disorders with musculoskeletal features Pattern of inheritance Musculoskeletal features Autosomal dominant Neurofibromatosis type 1 Scoliosis Short stature Neurofibromatosis type 2 Spinal tumour Achondroplasia Shortened limbs Facioscapulohumeral dystrophy Muscle weakness Marfan’s syndrome (see Case history, below) Tall stature, disproportionately long limbs, arachnodactyly, pectus excavatum/carinatum, pes planus Noonan’s syndrome Short stature, scoliosis, pectus, excavatum/carinatum, hypotonia Stickler syndrome (may also be autosomal recessive) Marfanoid habitus, scoliosis, joint hyperextensibility, spondyloepiphyseal dysplasia, premature osteoarthritis Cleidocranial dysostosis Absence of part or all of the clavicles, short stature Autosomal recessive Cystic fibrosis Arthralgia Inflammatory arthritis Hypertrophic pulmonary osteoarthropathy Mucopolysaccharidoses (except Hunter’s syndrome) Joint contractures, scoliosis, wide range of skeletal deformities Sickle cell anaemia Dactylitis, bone pain, avascular necrosis Homocystinuria Tall stature, arachnodactyly, pectus excavatum/carinatum, scoliosis X-linked Hypophosphataemic or vitamin D-resistant rickets Genu varum (bow legs) Duchenne muscular dystrophy Abnormal gait, muscle weakness, scoliosis, calf and deltoid muscle pseudohypertrophy Becker muscular dystrophy Abnormal gait, muscle weakness, toe walking, scoliosis Haemophilias A and B Haemophilia arthropathy Hunter’s syndrome (mucopolysaccharidoses) Stiffness, joint contractures, skeletal deformities (e.g. claw hand) Fabry’s disease Pain and burning sensation in hands and feet mutations. Skeletal abnormalities include marked short stature from shortening of the limbs, a large head, frontal bossing and depression of the nasal bridge. The hands are short and broad. A marked lumbar lordosis develops. • Hypotonia and delayed motor milestones are common. Other clinical features include hydrocephalus (1–2%), obstructive sleep apnoea and serous otitis media.
27 530Musculoskeletal disorders Toddlers learning to walk usually have flat feet due to flatness of the medial longitudinal arch and the presence of a fat pad, which disappears as the child gets older. An arch can usually be demonstrated on standing on tiptoe or by passively extending the big toe. Marked flat feet is common in children with hypermobility. However, in older children and young people, a rigid flat foot is pathological and is suggested by absence of a normal arch on tiptoeing; it may be due to an associated tendo Achilles contracture (ankle), or tarsal coalition (see below) or inflammatory arthropathy (JIA). Tarsal coalition results from lack of segmentation between one or more bones of the foot, and coalitions that were fibrous or cartilaginous become symptomatic as they begin to ossify. They become progressively more rigid and limit normal foot motion and often become symptomatic during the pre-adolescent years. In-toeing and out-toeing There are three main causes of in-toeing: • Metatarsus varus – an adduction deformity of a highly mobile forefoot. This occurs mainly in infants and is passively correctable. The heel is held in the normal position and no treatment is required unless it persists beyond 5 years of age. • Medial tibial torsion – at the lower leg, when the tibia is laterally rotated less than normal in relation to the femur. This occurs mostly in toddlers and may be associated with bowing of the tibia. It typically corrects by 5 years. • Persistent anteversion of the femoral neck – at the hip, when the femoral neck is twisted forward more than normal. This usually presents in early childhood and usually self-corrects by 8 years of age. It may be associated with hypermobility of the joints. Often children will sit between their feet with the hips fully internally rotated (‘W’ sitting). Out-toeing is uncommon but may occur in infants between 6 and 12 months of age. When bilateral, it is due to lateral rotation of the hips and resolves spontaneously. Toe walking is common in young children and may become persistent, usually from habit; they can walk normally on request. Alternatively, it may be due to mild cerebral palsy or tightness of the Achilles tendons and inflammatory arthritis in the foot and ankle (as in JIA). In older boys, Duchenne muscular dystrophy should be excluded. Pes cavus In pes cavus there is a high arched foot. When presenting in older children, it is often associated with neuromuscular disorders, e.g. Friedreich’s ataxia and type Musculoskeletal normal development Gait development There is considerable variation in the way normal gait patterns develop – these may be familial (e.g. ‘bottomshufflers’ often walk later) and subject to racial variation (e.g. Black African children tend to walk sooner and Asian children later than Caucasian children). The normal toddler has a broad base gait for support, and appears to be high-stepped and flat-footed with arms outstretched for balance. The legs are externally rotated with a degree of bowing. Heel strike develops around 15–18 months with reciprocal arm swing. Running and change of direction occur after the age of 2 years, although this is often accompanied by frequent falls until the child acquires balance and coordination. In the school-aged child, the step length increases and step frequency slows. Adult gait and posture occur around the age of 8 years. Normal gait follows ‘swing’, ‘stance’, and ‘toe-off’ phases; a painful or antalgic gait leads to shortening of the stance phase on the affected limb, and therefore lengthening of the swing phase. Leg alignment and feet position As children develop, there are a range of ‘normal’ abnormalities of leg alignment and feet position that generally are self-limiting and correct themselves over time (Table 27.3). In many children, leg alignment ‘evolves’ with initially a degree of leg bowing (in toddlers) followed by a degree of knock-knee (valgus) appearance. These changes usually resolve, with normal adult leg alignment being the case from around 8 years of age. The following are normal variants and most resolve without any treatment, but if painful, severe, progressive, or asymmetrical, should be referred for specialist opinion. Table 27.3 Normal variants of gait and posture seen in children Normal variants Normal age range Differential diagnoses to consider Bow legs 1–3 years Rickets, osteogenesis imperfecta, Blount’s disease Knock knees 2–7 years Juvenile idiopathic arthritis (JIA) Flat feet 1–2 years Hypermobility, congenital tarsal fusion In-toeing 1–2 years Tibial torsion, femoral anteversion Out-toeing 6–12 months Hypermobility, Ehlers–Danlos and Marfan’s syndromes Toe walking 1–3 years Spastic diplegia, muscular dystrophy, JIA
531 CHAPTER TWENTY-SEVEN suspected, physical examination is performed including an MSK examination (pGALS – paediatric Gait Arms Legs Spine; and pREMS – paediatric Regional Examination of the MSK System; see Further reading). Non-specific MSK pain in children is common and often labelled as ‘growing pains’; a confident diagnosis can be made when applying the ‘rules’ of growing pains (Table 27.4). Many children with non-specific aches and pains, including growing pains, are found to have joint hypermobility, although not all hypermobile children are symptomatic. In the child with hypermobility, rare but important syndromes need to be excluded (e.g. Marfan’s, Stickler’s, and Ehlers–Danlos syndromes), as these children are at risk of retinal and cardiac complications. Understanding applied science in musculoskeletal conditions The limping child The limping child is a common diagnostic problem. The differential diagnosis varies by age and is I hereditary motor sensory neuropathy (peroneal muscular atrophy). High fixed arches, pes cavus, and persistent toe walking may suggest neurological disease, but the latter has been reported as a feature of JIA. Clinical assessment of a child with musculoskeletal problems Young children may have difficulty in localizing or describing pain and the history is often given by the parent or carer or based on observations from others (e.g. teacher) and may be rather vague, with nonspecific complaints such as ‘my child is limping’. Symptoms such as pain, stiffness, decreasing ability (e.g. hand skills, handwriting, or sport), and reduced or altered interest in play activities may be observed and caregivers may have concerns about deterioration in behaviour (e.g. irritability, poor sleeping). Assessment of pain is important and may be conveyed through non-verbal signs such as withdrawal, crying, or distress. A delay in major motor development may indicate MSK problems as well as neurological disease. However, regression of achieved motor milestones is more likely in acquired MSK disease, such as muscle or joint disease; for example, the child who was happy to walk unaided but has recently refused to walk or resorted to crawling again. It is important to ask open questions and to enquire about mode of onset, site, distribution and nature of the symptoms and observations, features suggestive of multisystem involvement (e.g. rash, abdominal pain, headaches, Raynaud’s, fatigue), and red flags that warrant concern. It is often necessary to probe for symptoms of inflammatory joint or muscle disease (e.g. asking about the child’s mood, ‘gelling’ after periods of rest such as long car journeys, regression of achieved motor milestones, intermittent limping); a child will often adapt their activities to compensate for joint stiffness, pain, or weakness, and a change in the child’s play or reluctance to participate in activities may signify inflammatory joint or muscle disease. The features of inflammatory arthritis include joint swelling, warmth, loss of movement and tenderness on examination – an isolated hot red joint warrants mandatory investigation to exclude sepsis. However, in a well child with a monoarthritis, in the absence of trauma and sepsis, JIA is the most likely diagnosis; nonetheless mycobacterial infection must always be considered, especially in the immunocompromised or in endemic areas. The history alone may not identify sites of joint involvement and in all cases where MSK disease is Table 27.4 Features of growing pains and indications of concern ‘Rules’ of growing pains Pains never present at the start of the day after waking Child does not limp Physical activities not limited by symptoms Pains symmetrical in lower limbs and not limited to joints Physical examination normal (joint hypermobility may or may not be detected) Systemically well and major motor milestones normal Age range 3–12 years Indications for concern Systemic upset (red flags to suggest sepsis or malignancy) Abnormal growth (height and weight) Abnormal developmental milestones: Delay (especially major motor skills) suggestive of neurological disease or metabolic bone disease, OR Regression of achieved motor milestones (consider inflammatory joint or muscle disease) Impaired functional ability (ask about play, sport, schoolwork, ‘clumsiness’) Limping (intermittent or persistent) Morning symptoms (other than tiredness after disturbed sleep) or mood changes may suggest inflammatory arthritis Widespread pain (such as upper limbs and back) School absenteeism (Reproduced from Foster and Brogan (eds). Oxford Handbook of Paediatric Rheumatology. 2012.)
27 532Musculoskeletal disorders Table 27.5 Significant causes of limp, by age 0–3 years 4–10 years 11–16 years Most common Trauma (including toddler’s fracture) Trauma Transient synovitis Perthes disease Trauma Osgood–Schlatter disease Conditions requiring urgent intervention Osteomyelitis Septic arthritis Osteomyelitis Septic arthritis Osteomyelitis Septic arthritis Non-accidental injury Malignancy (e.g. neuroblastoma) Non-accidental injury Malignant disease (e.g. acute lymphocytic leukaemia) Slipped upper femoral epiphysis Malignancy (e.g. bone tumours) Testicular torsion Inguinal hernia Testicular torsion Appendicitis Inguinal hernia Testicular torsion Appendicitis Inguinal hernia Other important conditions to consider Developmental dysplasia of the hip JIA JIA JIA Metabolic (e.g. rickets) Haematological disease (e.g. sickle cell anaemia) Reactive arthritis Lyme arthritis (Reproduced from Foster and Brogan (eds). Oxford Handbook of Paediatric Rheumatology. 2012.) Question 27.2 Non-weight-bearing child with acute illness A 5-year-old boy presents with a 12-hour history of right leg pain and refusal to weight bear. He had been awake overnight complaining of severe, unremitting leg pain. On examination, he was flushed, tachycardic and had a temperature of 38.5°C. Which of the following is the most likely diagnosis? Select ONE answer only. A. Acute lymphoblastic leukaemia B. Developmental dysplasia of the hip C. Juvenile idiopathic arthritis D. Osteomyelitis E. Septic arthritis Answer 27.2 E. Septic arthritis. See below for explanation. haematogenous route and organisms are not always isolated. Septic arthritis and osteomyelitis may occur separately or together. The synovial membrane of the hip, ankle, shoulder and radial head inserts distally to the epiphysis in young children, allowing bacterial infection to spread directly from the bony metaphysis to the joint space. Around half of neonates with septic arthritis have concomitant osteomyelitis. This figure decreases to around one fifth in infancy and continues to decrease throughout early childhood. Sepsis from adjacent osteomyelitis is particularly common in the hip. Septic arthritis is most common in lower limb joints (knee>hip>ankle). Common causative organisms vary with age but include Staph. aureus, Group A streptococci and Gram-negative bacilli. Mycobacterial infection should be considered in the immunocompromised and in endemic areas. Septic arthritis usually presents with one or more of the following features: • Fever and associated constitutional symptoms • Joint pain, swelling, warmth and/or redness • Guarding of the affected area (e.g. refusal to weight bear) Signs and symptoms may be subtle and non-specific. Septic arthritis is a medical emergency, as bone and joint infections can be rapidly destructive. Delay in diagnosis or treatment can result in irreversible damage to the joint with associated long-term pain and functional impairment. Blood tests are not diagnostic of septic arthritis; they can only be used in the context of clinical findings. summarized in Table 27.5 and the accompanying clinical histories. Septic arthritis Septic arthritis is a bacterial infection of the joint, most common in the first two years of life. Osteomyelitis is an infection of bone, again most common under two years of age, and may be acute, sub-acute or chronic. Most bone infections are spread via the
533 CHAPTER TWENTY-SEVEN frequently the first-line antibiotic of choice. Total duration of treatment is controversial, but children are commonly given 10–14 days of IV antibiotics before switching to oral preparation if response is adequate. Transient synovitis Transient synovitis is the most common cause of hip pain in children between three and ten years of age and is more common in boys. It is a diagnosis of exclusion. Little is known about the aetiology of this condition, although viral, autoimmune and allergic associations have been suggested. Around half of children with transient synovitis report a viral infection during the preceding week. A painless limp in a relatively well child is the most common presentation; high fever, markedly elevated inflammatory markers, severe pain or functional impairment suggest an alternative diagnosis. Hip X-ray should be normal but ultrasound scan (USS) of the hips may reveal a small effusion. Most cases of transient synovitis settle quickly. Review is necessary if the limp persists to exclude Perthes disease (or SCFE in older children) or evolving JIA. Case history Limp and normal blood tests A four-year-old girl presented to her local walk-in centre with a four-day history of left leg pain and reluctance to walk. She had been off nursery with a viral infection five days earlier but was now relatively well in herself, playing happily if allowed to remain seated. She was afebrile and systems examination was unremarkable. Left hip had a reduced range of active and passive movement compared to the right. Blood tests (FBC, CRP and ESR) and hip X-ray were normal. In the absence of ‘red flag’ features suggestive of serious underlying illness, she was diagnosed with transient synovitis of the left hip and allowed home with instructions to return if symptoms did not settle completely within the next few days. Case history Chronic history of leg pains and poor growth A four-year-old girl presented to rheumatology clinic with a several month history of bilateral leg pain, worse towards the end of the day, associated with an increased frequency of falls. She was a previously healthy child of South Asian origin with no family history of note. She was the youngest of three children, all of whom had been exclusively breastfed to six months. The child appeared generally healthy, although clingy, and was noted to be on the 0.4th centile for weight and height. There was no suggestion of synovitis and muscle strength was normal throughout. Genu varum (‘bowing’ of the legs) was noted. Bloods tests revealed normal inflammatory markers and a mild hypochromic anaemia. The serum alkaline phosphatase was markedly elevated with low/normal serum calcium and phosphate. 25-hydroxyvitamin D was 18 nmol/L (normal >50 nmol/L, insufficient 25–50 nmol/L and low <25 nmol/L). X-ray of the legs demonstrated widened growth plates and metaphyseal fraying of the long bones with marked genu varum. Wrist X-ray revealed cupping and fraying of the metaphyseal region. Chest X-ray was normal. Kocher proposed four factors to predict septic arthritis: • Fever >38.5°C • Non-weight bearing or pain with passive motion of the joint • ESR >40 mm/hour (a raised CRP may be used as an alternative) • White blood cell count >12 × 109 Therefore, helpful investigations include: • Full blood count (FBC), erythrocyte sedimentation rate (ESR), C-reactive protein (CRP): Children may have markedly elevated neutrophil and platelet counts. CRP is a better predictor than ESR for acute infection (see ‘Inflammatory markers’, below). • Blood cultures: Not always positive in septic arthritis and frequently negative in osteomyelitis. • Joint aspiration with Gram stain microscopy and culture of synovial fluid. • X-ray: May be normal in the early stages of infection. Bony changes can take up to 21 days to evolve. • Ultrasound: Can be useful in septic arthritis to demonstrate joint effusion and guide aspiration. • Magnetic resonance imaging (MRI): Useful if diagnosis unclear. Radionuclide bone scanning is an alternative if MRI is unavailable. Management requires appropriate antibiotics and should be in conjunction with the local microbiology team. Cefuroxime, a broad spectrum second-generation cephalosporin with good bone penetration, is
27 534Musculoskeletal disorders may result from low exposure to ultraviolet light rays, nutritional deficiency, liver or kidney disease. Breastfed infants whose mothers are not exposed to sunlight or who themselves are not exposed to sunlight are at higher risk. The incidence of vitamin D deficiency in the UK has increased in the last few years with cases reported in children of all ethnic origins. One putative association is the widespread use of sunblock, preventing ultraviolet light from reaching the surface of the skin. The definition, prevention and treatment of vitamin D deficiency in the UK remains controversial (see Chapter 13, Nutrition). The Royal College of Paediatrics and Child Health suggests treatment doses of between 3000 and 10,000 U daily, depending on the age of the child, for 4–8 weeks. Question 27.3 Rickets A 3-year-old girl presents to the emergency department with a short history of abdominal pain and muscle weakness. On examination she is small (<2nd centile for weight and height) with frontal bossing and bowing of the legs. You suspect she may have rickets secondary to vitamin D deficiency. Are the following statements about vitamin D deficiency true (T) or false (F)? A. 25-hydroxylation of vitamin D occurs in the skin B. Breast milk contains more vitamin D than formula milk C. Calcidiol is the biologically active form of vitamin D D. It is more common in infants born to mothers with higher vitamin D levels E. Typically less than 10% of vitamin D comes from dietary sources Answer 27.3 A. False; B. False; C. False; D. False; E. True. See below for discussion. Vitamin D deficiency rickets Rickets is a defective ossification of the bony matrix and can be due to deficiency of the active form of vitamin D (1, 25-dihydroxyvitamin D or 1, 25-vitamin D), deficiency of phosphate or, rarely, deficiency of calcium. Signs and symptoms include bone tenderness, joint pain, proximal muscle weakness, delayed dentition, increased frequency of fractures, growth delay and skeletal deformities. Skeletal deformities include bowing of the long bones and persistent genu varum, splaying of the rib cage and costochondral swelling (the so-called ‘rachitic rosary’). X-rays reveal impaired mineralization of the growth plates with cupping and fraying of the margins of the metaphyses. Vitamin D deficiency is the commonest cause of rickets in the UK and worldwide. The normal source of 1,25-vitamin D is the skin (which is responsible for production of 90% of bioavailable vitamin D in temperate regions). In the skin, ultraviolet light rays convert 7-dehydroxycholesterol into the vitamin D prohormone. The prohormone is converted to 25-hydroxyvitamin D (calcidiol) in the liver and then to 1,25-dihydroxyvitamin D in the kidneys. Cutaneous vitamin D production is higher in pale-skinned individuals. Deficiency of 1,25-dihydroxyvitamin D Case history Joint swelling and intermittent limp A previously healthy seven-year-old girl presented to her general practitioner with a seven-week history of right knee swelling and intermittent limp. She was stiff and grumpy in the mornings but otherwise well in herself. Her teacher had reported that she was finding it difficult to sit cross-legged in assembly. She was systemically well with no ‘red flags’ in the history. If a limp persists for >3 weeks, the likelihood of JIA is high. It is advisable to refer to paediatric rheumatology prior to invasive procedures (arthroscopy or MRI) as such investigations are usually not necessary to confirm the diagnosis. Although there is no diagnostic test for JIA, further investigation must exclude alternative diagnoses such as Perthes disease, chronic infection or malignancy. JIA is a clinical diagnosis and blood tests (FBC, ESR, CRP) can be normal and rheumatoid factor is invariably negative. Eye screening is essential if JIA is suspected as chronic anterior uveitis, if present, is usually asymptomatic. This child was referred to her local paediatric rheumatology team and diagnosed with JIA. Arthritis in children and young people is invariably a result of inflammatory pathways and likely triggered by a combination of genetic and environmental factors. JIA is the commonest cause of chronic arthritis in children (incidence 1 in 10,000 per year, prevalence 1 in 1000) and encompasses a heterogeneous group of diseases of unknown aetiology. Without treatment, inflammatory arthritis can result in joint damage and disability along with impact on growth, both localized (such as leg length discrepancy or micrognathia) through to generalized growth retardation from chronic disease and compounded by use of corticosteroids.
535 CHAPTER TWENTY-SEVEN children have normal blood counts despite obvious inflammation on clinical examination. In extensive polyarticular disease, systemic-onset juvenile idiopathic arthritis (SOJIA) or connective tissue disease (such as SLE), marked thrombocytosis and leukocytosis (predominantly polymorphs) can be seen. The serum ferritin is an acute phase reactant and can be markedly elevated in systemic onset JIA and SLE. A sudden dramatic rise in the serum ferritin in association with low or falling haematological indices can herald the onset of macrophage activation syndrome (MAS), a severe and potentially life-threatening complication of SOJIA and other chronic rheumatic diseases of childhood. Unexpectedly low/normal platelet or white cell counts in children with marked systemic inflammation suggest the possibility of occult malignancy (for example, leukaemia or lymphoma). A normocytic normochromic anaemia of chronic illness can develop in chronic inflammatory disease and iron deficiency is common in children with long-standing or severe disease. Autoimmune haemolytic anaemia can complicate systemic inflammatory illnesses such as SLE. Inflammatory markers Inflammatory markers can be useful measures of disease activity. The C-reactive protein (CRP) is an acute phase protein, synthesized by the liver in response to activated macrophages. Its physiological role is to bind to phosphocholine, expressed on the surface of dead or dying cells, thereby activating the complement system. The CRP rises very rapidly in response to acute inflammation such as infection, rising to above normal by six hours and peaking by around 48 hours. The level is determined by the rate of production or the severity of the inflammatory process. The erythrocyte sedimentation rate (ESR) is a measure of the rate of fall of erythrocytes in anticoagulated blood during a one-hour period (mm/hour). During inflammation, relatively high levels of fibrinogen cause erythrocytes to form rouleaux (chains or stacks of erythrocytes), which settle more quickly. The ESR is therefore a reliable measure of systemic inflammation. ESR and CRP appear to be equally reliable as an initial screening test but there are important differences in rates of increase and subsequent reduction. The CRP is rapidly responsive to resolution of inflammation (particularly relevant in the case of acute infection) but the ESR will rise more slowly and remain elevated for a longer period of time following inflammation. In the absence of a definitive gold standard assay of disease activity, the ESR is frequently used to assess disease activity at presentation and during follow-up of Question 27.4 Investigations Below (A–I) is a list of possible diagnoses: A. Churg–Strauss syndrome B. Granulomatosis with polyangiitis (Wegener’s granulomatosis) C. Juvenile dermatomyositis D. Juvenile idiopathic arthritis E. HIV F. Scleroderma G. Sjögren’s syndrome H. Systemic lupus erythematosus (SLE) I. Systemic sclerosis Match the diagnosis with the blood results below. Each answer may be used once, more than once, or not at all. Elevated ESR and CRP with: 1. 1 in 160 titre positive antinuclear antibody (ANA), 1 in 80 titre positive anti-double stranded DNA antibody (ds-DNA) 2. 1 in 320 titre positive anti-neutrophil cytoplasmic antibodies (cANCA) 3. Positive rheumatoid factor and anti-CCP antibodies Answer 27.4 1. H. Systemic lupus erythematosus (SLE). 2. B. Granulomatosis with polyangiitis (Wegener’s granulomatosis). 3. D. Juvenile idiopathic arthritis. See below for discussion. Investigations Many MSK diseases do not have ‘gold standard’ diagnostic tests or simple, reliable measures of disease activity and instead require a high index of suspicion in the context of the history and examination. Patients may require multiple laboratory and radiological examinations, the results of which have to be pieced together like a jigsaw puzzle. Laboratory investigations The roles of the most commonly requested laboratory examinations are discussed in more detail below. Full blood count and serum ferritin The haematological indices can suggest an infective or inflammatory process. However, in JIA many
27 536Musculoskeletal disorders connective tissue disease, Sjögren’s syndrome, juvenile dermatomyositis, scleroderma and systemic sclerosis. A positive ANA can occur in some children with JIA, most frequently girls with younger age at onset and an oligoarticular disease course. Children with oligoarticular JIA who are ANA positive are at highest risk for chronic anterior uveitis, a potentially devastating complication of this illness. Anti-double stranded DNA antibodies (dsDNA) dsDNA autoantibodies are highly specific for SLE and are seen in the majority of children with lupus nephritis. Titres correlate with disease activity in some children with SLE. Other extractable nuclear antigens (including anti-RNP antibodies, anti-Sm antibodies, anti-Ro and anti-La antibodies) occur with variable frequencies in children with SLE and related connective tissue disorders. Anti-phospholipid antibodies This is a heterogeneous group of autoantibodies which bind to phospholipids in the cell membrane. Examples include anticardiolipin antibodies and β2- glycoprotein 1 antibodies. Antiphospholipid antibodies may occur in primary antiphospholipid syndrome, SLE and some vasculitides. They can be found in association with viral infections and may be drug induced. Not all antiphospholipid antibodies are pathogenic and are known to occur in up to 8% of the normal adult population. In view of this, antiphospholipid antibodies should only be considered positive if present on at least two occasions at least three months apart. Children with persistently positive antiphospholipid antibodies should be commenced on prophylactic low-dose aspirin to modify the risk of thromboembolic phenomena. Anti-neutrophil cytoplasmic antibodies This group of antibodies target lysosomal enzymes (myeloperoxidase and proteinase 3) in neutrophils and monocytes. There are two main patterns of immunofluorescent staining: perinuclear (pANCA) and cytoplasmic (cANCA). About 90% of cANCA is directed against proteinase 3 (PR3) and around 70% of pANCA against myeloperoxidase. PR3-ANCA is commonly found in granulomatosis with polyangiitis (previously known as Wegener’s granulomatosis) and can be found in microscopic polyangiitis. MPO-ANCA is typically associated with microscopic polyangiitis or occasionally with Churg–Strauss syndrome. ANCAs are not absolutely specific for the ANCAassociated vasculitides. They can be detected in chronic infection (including tuberculosis, the viral hepatides and HIV), malignancy, inflammatory bowel disease, sclerosing cholangitis and can be drug-induced. children with rheumatic illnesses. The ESR is central to the majority of composite indices of disease activity developed for paediatric rheumatic illnesses, for example the Juvenile Arthritis Disease Activity Score (JADAS). A sudden drop in the ESR can herald the onset of MAS in children with SOJIA or SLE. As the CRP increases more rapidly than the ESR in response to systemic inflammation it is more readily confounded by the presence of infection. Immunosuppressed children, particularly children using interleukin-6 blockade, may not mount a normal immune response to infection. Acute phase reactants such as the CRP must be interpreted with caution in this situation. Serum immunoglobulins and the C4 complement factor reflect the acute phase reactants and can be markedly elevated in children with very active inflammatory disease. Low C3 or C4 complement factors suggest active SLE. Rheumatoid factors and anti-cyclic citrullinated protein (anti-CCP) antibodies Classic rheumatoid factors (RF) are IgM antibodies directed against human IgG. The significance of RFs of other immunoglobulin isotypes is uncertain. RF is not a diagnostic test for JIA. In contrast to adults, less than 10% of children with JIA are RF positive. RF can be elevated as an acute phase reactant (for example in bacterial endocarditis), so a result can only be considered positive if the RF is present in high titre on at least two occasions. RF can be associated with other rheumatological illnesses of childhood, in particular SLE and the overlap conditions such as mixed connective tissue disease. Anti-cyclic citrullinated protein (anti-CCP) antibodies are less prevalent in children with JIA than in adults with rheumatoid arthritis, but occur in children with RF-positive polyarticular JIA. Anti-CCP antibodies are thought to be associated with an increased risk of erosive (and therefore potentially disabling) joint disease. Antinuclear antibodies Autoantibodies are immunoglobulins produced in response to self-antigens. Antinuclear antibodies (ANAs) are directed against the nuclear contents of the cell. Titres are usually reported as positive between 1:40 and 1:80. ANAs are not a useful screening tool for autoimmune disease and results must be interpreted in the context of clinical findings. A positive ANA can be found in up to 15% of healthy children and can occur as a consequence of viral infection, malignancy or IgA deficiency. A persistently positive ANA can be associated with a number of rheumatological conditions, including SLE, drug-induced lupus, undifferentiated
537 CHAPTER TWENTY-SEVEN with suspected juvenile dermatomyositis and has supplanted muscle biopsy in diagnosis and disease monitoring. Other investigations Synovial fluid examination to exclude sepsis is mandatory in the assessment of a child with a single hot, swollen joint. Mycobacterial infection can be indolent and easily missed and diagnosis may require further examination of the synovial fluid or tissue culture, although polymerase chain reaction (PCR), if available, can give quicker results. Muscle enzymes, bone chemistry and other indicators of systemic disease may be required, as well as genetic investigations for conditions with known mutations. Investigations in clinical practice These are outlined in the case histories below. Autoantibodies in systemic sclerosis The autoantibody profile can predict the disease pattern in some children with complex multi-system connective tissue disease. Systemic sclerosis is a multisystem autoimmune disease characterized by increased fibroblastic activity. Autoantibodies can predict the extent of skin and other organ involvement. Antitopoisomerase 1 (Scl 70) antibody occurs in systemic sclerosis and is associated with lung fibrosis and renal disease. Anti-PM-Scl antibody is associated with a combination of myositis and scleroderma. Anti-U1RNP antibody is associated with arthritis and overlap syndromes. Anti-centromere antibody is found in limited cutaneous systemic sclerosis and is associated with an increased risk of pulmonary hypertension. Anti-RNA antibody is associated with diffuse cutaneous systemic sclerosis. Radiological examination Plain X-ray can be used in the initial investigation of children with bone pain. Classical appearances can be seen, e.g. Perthes disease (irregularity of the femoral head) and SCFE (displaced femoral head). The presence of periosteal reaction suggests the possibility of bony infection or malignancy. An AP view of the hand and wrist can be a useful screening tool in the investigation of metabolic bone disease (for example, rickets). If a skeletal dysplasia is suspected, specific skeletal radiographic survey may be performed. X-rays may be normal in early JIA. Although plain X-rays may reveal soft tissue swelling around joints or demonstrate bony erosions, they are insensitive for the diagnosis of acute synovitis. Ultrasound Musculoskeletal ultrasound can help to identify which structures are involved in a clinically swollen joint. Ultrasound can, with specialist interpretation, differentiate between joint effusions, synovial hypertrophy, synovitis, enthesitis, tendonitis and tenosynovitis. It is cheap, more readily available and is well tolerated by most children. Magnetic resonance imaging Magnetic resonance imaging (MRI) allows the localization and differentiation of bony and soft tissue lesions. It provides an anatomical overview and can detect inflammation, damage and pathology such as tumour. MRI is sensitive to early changes in JIA. However, MRI is not always available and requires sedation or general anaesthesia in younger children. Gadolinium contrast may be indicated to identify if inflammatory disease is present. MRI scanning of the thigh muscles can be diagnostic in children Case history Investigation of suspected JIA A two-year-old girl is referred to your clinic with an eight-week history of swollen right knee. Her parents have noticed that she is asking to be carried more often but she remains generally well. What would be the appropriate initial investigations? This little girl is likely to have oligoarticular JIA, but initial investigations must exclude other possible diagnoses, including septic arthritis and malignancy. Investigations should include: FBC: This should be normal in a child with one swollen joint. Low indices should raise the possibility of haematological malignancy, such as acute lymphoblastic leukaemia and further investigations including blood film and bone marrow aspirate may be required. Significantly elevated neutrophil or platelet count may indicate underlying infection and further investigations including blood cultures and synovial fluid aspiration may be required. Acute phase reactants: Significantly elevated acute phase reactants are unusual in oligoarticular JIA and should trigger further investigations to exclude infection and/or malignancy. Anti-nuclear antibody: A positive ANA can occur in JIA and is associated with an increased risk of uveitis. Although all children with JIA are at risk from uveitis, young children, particularly girls, with persistently positive ANA are at higher risk. Rheumatoid factor: RF is unlikely to be positive in children with oligoarticular JIA.
27 538Musculoskeletal disorders Treatment of inflammatory musculoskeletal conditions Immunopharmacology treatment HLA B27: There are HLA associations for each JIA subtype, particularly important in oligoarticular disease. HLA B27 plays an important role in the classification of JIA and is associated with both enthesitis-related arthritis and psoriatic arthritis. HLA B27 positivity predicts more aggressive disease. Plain XR: May be indicated in children with long-standing disease to delineate the presence of erosive damage. US: Can be very useful in determining the presence of sub-clinical synovitis and/or tenosynovitis. MRI: May be useful in children with unusual presentations or long-standing disease in which it can be difficult to differentiate between damage and active synovitis. Slit-lamp examination: Chronic anterior uveitis affects one third of children with JIA, is invariably asymptomatic in early stages, and can result in blindness if not diagnosed and treated. Slit-lamp examination is essential for diagnosis. Case history Investigation of suspected connective tissue disease A 14-year-old girl is referred urgently to clinic with a six-month history of worsening fatigue and arthralgia, complicated by recent-onset dry cough and epistaxis. What would be the appropriate initial investigations? This girl is systemically unwell and the differential diagnosis has to include chronic infection (for example, tuberculosis), connective tissue disease, vasculitis and malignancy. Investigation of complex multisystem illness is initially non-specific and early results will determine further, disease-specific investigations: FBC: A normochromic, normocytic anaemia is typical of chronic illness, although a hypochromic picture can also occur. Neutrophils and platelets may be normal or elevated in connective tissue disease, reflecting the underlying inflammatory process. Low white cells or platelets should raise the suspicion of malignancy. Acute phase reactants: Acute phase reactants are often elevated in connective tissue disease (CTD), although can be normal in vasculitis. Renal function and urinalysis: Renal involvement is common in connective tissue disease (particularly in SLE) and vasculitis (particularly Henoch–Schönlein purpura, granulomatosis with polyangiitis, microscopic polyarteritis and Churg–Strauss syndrome). Renal function is usually preserved until late in the illness, but microscopic haematuria and proteinuria are common early manifestations of nephritis. Urine dipstick is therefore integral to the initial investigation of a child with suspected CTD/ vasculitis and any abnormalities should trigger a request for renal biopsy. Liver function: Liver enzymes are commonly elevated in the early phases of connective tissue disease/ vasculitis. The differential diagnosis must include chronic viral infection and malignancy. Complement and immunoglobulins: Immunoglobulins can be elevated in any chronic inflammatory process. Low C4 is a consistent and reliable indicator of lupus nephritis. Autoantibodies: Autoantibodies can be very helpful in the investigation of suspected connective tissue disease/vasculitis, but must be interpreted in the context of the clinical picture. The ANA is very non-specific, but is frequently positive in children with CTD/vasculitis. Antibodies to dsDNA are virtually pathognomonic of SLE and may occur in combination with anti-Ro and anti-La antibodies (associated with neonatal lupus erythematosus and Sjögren’s syndrome) and anti-Sm antibodies. Anti-RNP antibodies are found in high titres in children with mixed connective tissue disease. Echocardiogram: Pericardial effusions, reduced left ventricular function and myocardial involvement can all complicate multisystem CTD/vasculitis. USS abdomen and pelvis: Important to exclude intra-abdominal infection or masses. May demonstrate organomegaly or serositis. High-resolution CT chest: Pulmonary involvement is a potentially devastating consequence of a number of CTD/vasculitides, including granulomatosis with polyangiitis, Churg–Strauss syndrome and systemic sclerosis. Angiography: Angiography is central to the diagnosis of vasculitis. CT/MR angiography are very useful and relatively safe procedures and can be used in place of conventional angiography. Case history A ten-year-old girl came home from school complaining of some discomfort in both her knees. Bilateral knee swelling was noted by her GP and regular ibuprofen advised. While this eased her pain, morning stiffness became problematic and she was referred to her local paediatric rheumatology
539 CHAPTER TWENTY-SEVEN administration include oral, intra-articular, intravenous, intramuscular and topical (e.g. for skin and eyes). If a child is taking oral corticosteroids, dietary intake of calcium and vitamin D should be optimized; however, the role of calcium and vitamin supplements to reduce the risk of osteoporosis is unclear. Disease-modifying anti-rheumatic drugs Methotrexate (MTX) is the most widely used diseasemodifying anti-rheumatic drug (DMARD) in the treatment of JIA and is often used in other inflammatory diseases. It is a structural analogue of folic acid, which competitively inhibits dihydrofolic acid, binding to the enzyme dihydrofolate reductase. The amount of intracellular folinic acid (the active metabolite of dihydrofolic acid) is decreased, and affects the intracellular folinic acid-dependent metabolic pathways. Purine and pyrimidine metabolism are two such pathways. Whilst these pathways are considered important, the exact mechanism of action of MTX in these conditions remains unclear. Other DMARDs, including sulphasalazine, azathioprine and ciclosporin, may be useful. In the child with severe disease resistant to or intolerant of MTX, biological therapies are often used (see below), although in many parts of the world, access to these is very limited due to their high cost. Biologic and novel therapies A major advance in the management of many inflammatory conditions has been the advent of ‘cytokine modulators’ or ‘biologics’, to block selectively the effects of pro-inflammatory cytokines. Anti-TNF treatment has been a major advance in the management of JIA. Table 27.6 summarizes the cytokine modulators which have been used in severe JIA, although not all are licensed for use in children, and Box 27.1 Non-steroidal anti-inflammatory drugs Non-steroidal anti-inflammatory drugs (NSAIDs) act by inhibiting enzymes (cyclo-oxygenase-1 (COX-1) and COX-2) involved in prostaglandin synthesis (Fig. 27.3), resulting in analgesic, anti-inflammatory and antipyretic effects. There are two broad groups of NSAIDs – the older, traditional, non-selective NSAIDs (e.g. ibuprofen, piroxicam, diclofenac, aspirin) that inhibit both COX-1 and COX-2 and the newer, selective COX-2 inhibitors that predominantly inhibit COX-2 (e.g. celecoxib). The clinical effects of NSAIDs depend largely on their selectivity for these enzymes. The non-selective group act widely, resulting in varying degrees of analgesic, anti-inflammatory, antipyretic and antiplatelet effects, whereas the selective group has fewer gastrointestinal tract side effects. NSAIDS may provide some symptom relief initially from their anti-inflammatory effect, although they are not disease-modifying and do not prevent joint damage in JIA. The last decade has seen a marked change in treatment with the emergence of potent immunosuppressive agents used early in the disease course to optimize long-term outcomes. Corticosteroids These are potent immunosuppressants but their use is limited by troublesome side-effects and the risk of iatrogenic adrenal suppression. Common routes of Fig. 27.3 Enzymes cyclo-oxygenase-1 (COX-1) and COX-2 are involved in prostaglandin synthesis. NSAIDs act by inhibiting the enzymes. Phospholipid Arachidonic acid Cyclo-oxygenase -1 and 2 Intermediate prostaglandin Prostacycline Vasodilation Platelet aggregation inhibition Prostaglandins Inflammation, pain, fever, gastroprotection Thomboxane A2 Platelet aggregation department, where she was diagnosed with JIA. Initial treatment involved intra-articular steroid injections to both knees. Six months later, she was commenced on subcutaneous methotrexate due to active arthritis in multiple joints (ankles, elbows and wrists). Due to persisting disease, etanercept was added after 3 months resulting in good disease control and no active disease.
27 540Musculoskeletal disorders collect data to address the potential theoretical concerns about infection risk, impact on fertility and malignancy risk. The use of biological therapies in JIA-related uveitis is increasing and clinical trials are in progress. For those few children with severe refractory disease failing to respond to cytokine modulators, a further option is T-cell depletion coupled with autologous haematopoietic stem-cell rescue; this procedure is limited to specialist centres and needs careful selection of patients. Periodic fever syndromes and autoinflammatory disease Hereditary periodic fever syndromes have onset usually in early childhood and are associated with genetic mutations. Many of the syndromes (Table 27.7) have mutations described and diagnostic tests and specific highly effective biological treatments now available. Others, such as PFAPA syndrome (periodic fever, aphthous stomatitis, pharyngitis, and cervical adenitis), Behçet’s disease and chronic recurrent multifocal osteomyelitis (CRMO), have less clear aetiologies but have clinical features that suggest they are likely to be autoinflammatory. These disorders of innate immunity are characterized by recurring episodes of fever and constitutional upset, but the child is often well between attacks. Other systemic inflammatory symptoms are variable and often include joints, skin, eyes, serosa and central nervous system with high morbidity. High acute phase reactants are typical along with leukocytosis. Long term, if not treated, they can result in AA amyloidosis with high mortality. summarizes the important issues for children taking cytokine modulators. Emerging evidence demonstrates dramatic and sustained improvement with reduced joint damage in children with JIA who had failed MTX due to ongoing disease or intolerance (nausea and vomiting most common). Long-term safety and efficacy data will be obtained through registries and international collaboration is necessary to Table 27.6 Cytokine modulators currently used in rheumatic disease Generic name Mechanism of action Route of administration Etanercept TNF-α soluble receptor that binds to circulating TNF and competes with membrane receptor Subcutaneous injection Infliximab Human–murine chimeric antibody that neutralizes TNF-α Intravenous infusion. Adalimumab Fully human monoclonal antibody that neutralizes TNF-α Subcutaneous injection Anakinra Rilonacept Canakinumab IL-1 receptor antagonists Subcutaneous injection Rituximab Human–murine chimeric antibody against CD20 (depletes B cells) Intravenous infusions Tocilizumab IL-6 receptor antagonist Intravenous infusion Abatacept CTLA4-antagonist to block T cell and B cell interaction and initiation of the pro-inflammatory pathway Intravenous infusion *These apply to all cytokine modulators. Box 27.1 Important issues for the child taking cytokine modulators* • Avoid live viral vaccines • Promote annual flu vaccine • Promote pneumococcal immunization (current advice: 5-yearly) • Be vigilant regarding infections (e.g. varicella and shingles, opportunistic infections such as listeriosis). Vaccinations against varicella advised if there is a window of time; specialist advice needed. • Advice regarding travel abroad, with medicines and travel insurance Case history Use of biologics in periodic fever syndrome A 2-year-old boy was referred with history of fatigue, frequent fever episodes and muscle pain. He developed an urticarial rash during the first few months of his life and had been admitted with two episodes of presumed meningitis (cultures were negative). He subsequently developed swellings of both knees. He was investigated by paediatric rheumatology and clinical genetics and found to have a mutation in NLRP3. He was treated initially with anakinra but struggled with daily subcutaneous injections. He subsequently received monthly canakinumab with marked improvement in his well-being and reduction in his episodes of fever and rash. Cryopyrin-associated periodic syndrome (CAPS) is subdivided into familial cold autoinflammatory syndrome (FCAS), Muckle–Wells syndrome (MWS) and chronic infantile, neurological, cutaneous and
541 CHAPTER TWENTY-SEVEN Summary We have described the spectrum of MSK diseases and disorders, with their link to basic science as appropriate. Many MSK conditions do not have diagnostic tests and the case histories highlight the importance of MSK assessment as an integral part of general paediatrics, with a need to interpret clinical examination and investigations within their clinical context. Further reading Foster HE, Brogan PA. Oxford handbook of paediatric rheumatology. Oxford: Oxford University Press; 2012. Newcastle University and Northumbria University. Paediatric musculoskeletal matters (PMM), <http:// www.pmmonline.org>; 2015 [accessed 31.08.15]. A free online resource with a wealth of information about musculoskeletal medicine, including video demonstrations of joint examination (pGALS, pREMS), and cases. articular syndrome/neonatal onset multisystem inflammatory disease (CINCA/NOMID). CAPS is associated with mutations in NLRP3/CIAS1 on chromosome 1q44, and such mutations result in IL-1 activation. Disease onset is in early infancy and features may be present from birth. Inheritance is autosomal dominant (75% of FCAS and MWS patients). CINCA however often results from new mutations. FCAS typically presents with bouts of urticarial rash, fever, arthralgia and conjunctivitis and is often triggered by damp or cold conditions. MWS typically presents with fever, usually daily (afternoon/evening), along with arthralgia, myalgia, urticarial rash (which may be persistent), conjunctivitis and malaise. Deafness develops later. CINCA/NOMID – clinical features result from chronic widespread inflammation – headaches, raised intracranial pressure, optic atrophy, uveitis, deafness, developmental delay, joint damage. IL-1 blockade is an effective treatment for CAPS. Table 27.7 Periodic fever syndromes Periodic fever syndrome Gene Mode of inheritance Typical duration of attacks Typical frequency of attacks Characteristic laboratory abnormalities Treatment Familial Mediterranean fever MEFV Chromosome 16 Autosomal recessive (dominant in rare families) 1–3 days Variable Marked acute phase response during attacks Colchicine TRAPS (tumour necrosis factor alpha receptor-1 associated syndrome) TNFRSF1A Chromosome 12 Autosomal dominant, can be de novo More than a week, may be very prolonged Variable, may be continuous Marked acute phase response during attacks. Low levels of soluble TNFR1 when well Etanercept High-dose corticosteroids with attacks Anti-IL-1 therapies HIDS (hyperimmunoglobulin D syndrome) MVK Chromosome 12 Autosomal recessive 3–7 days 1–2 monthly Elevated IgD and IgA, acute phase response, and mevalonate aciduria during attacks Anti-TNF and antiIL-1 therapies Anti-IL-1 therapies CAPS (cryopyrin-associated periodic syndrome) NLRP3 Chromosome 1 Autosomal dominant or sporadic Continuous, often worse in the evenings Often daily Varying but marked acute phase response most of the time Anti-IL-1 therapies PAPA – Pyogenic arthritis, pyoderma gangrenosum and acne PSTPIP1 (CD2BP1) Chromosome 15 Autosomal dominant Intermittent attacks with migratory arthritis Variable, and may be continuous Acute phase response during attacks Anti-TNF therapy or Anti-IL-1 therapies DIRA (deficiency of lL-1-receptor antagonist) IL1RN Chromosome 2 Autosomal recessive Continuous Continuous Marked acute phase response IL-1Ra Blau syndrome NOD2 (CARD15) Chromosome 16 Autosomal dominant Continuous Continuous Sustained modest acute phase response Immunosuppressive therapies TNFR1, tumour necrosis factor alpha receptor-1 gene. (Modified from Foster and Brogan (eds). Oxford Handbook of Paediatric Rheumatology. 2012.)
This page intentionally left blank
LEARNING OBJECTIVES By the end of this chapter the reader should: • Know the anatomy and embryology of the central and peripheral nervous systems • Understand the role of cerebrospinal fluid (CSF) and how CSF analysis can be used for diagnosis of neurological conditions including infections • Understand how the physiology of the central and peripheral nervous systems applies to neurophysiology (e.g. EEG, EMG) • Understand the role of imaging in neurological disorders • Know the genetic and environmental factors in the aetiology of important neurological disorders and brain development • Understand the physiological and pathophysiological changes that occur in neurological disorders, including headache, migraine, raised intracranial pressure and epilepsy • Know the pharmacology of agents commonly used in neurological disease • Understand the basis of non-pharmacological treatments for the management of neurological disorders 543 CHAPTER TWENTY-EIGHT Development of the central nervous system During the third week of gestation, the three germ layers of the embryo are formed (ectoderm, mesoderm and endoderm; Fig. 28.1). The central nervous system appears at the beginning of the third week, being derived from the primitive streak. The cephalic end of this streak is known as the primitive node and surrounds the primitive pit. This process is called gastrulation. From the 3rd to 8th weeks, each of the three germ layers give rise to specific tissues and organs. This is the embryonic phase, following which the shape of the embryo changes markedly; the major external features are recognizable by the end of the second month. Inside-out development Primitive neural tissue arises from the ectoderm. Maturation of the primitive pit leads to the neural plate and neural folds. This is the basis of the nervous system. The neural folds become elevated and grow towards the midline to fuse and thus form the neural tube. The process of fusion begins in the cervical region and progresses both in the cephalic and caudal directions, with full closure being achieved at 25–27 days. The caudal end of the neural tube is composed of neuroepithelial cells, which give rise to primitive nerve cells known as neuroblasts, which following maturation form the grey and white matter of the future spinal cord. The cephalic end progresses through a process of folding to form the primitive hindbrain, midbrain, and forebrain. The hindbrain, midbrain, and forebrain have distinct but interlinked functions, ranging from the hindbrain, which controls basic processes necessary to sustain life, to the intricacies of cerebral cortex activity in higher functions, such as completing a jigsaw puzzle or playing a musical instrument. Development in general begins with the ‘primitive’ brain and works upwards to the cortex. Therefore, it progresses first in Gary McCullagh, Dipak Ram, Nadya James Neurology C H A P T E R 28
28 544Neurology • Cranial defects – anencephaly, where there is failure of development of most of the cranium and cerebral hemispheres • Midline defects – failure of fusion, e.g. of skull in encephaloceles, most often occipital, with herniation of meninges and may include neural tissue • Spina bifida – by definition involves splitting of the vertebral arch, and may involve the spinal cord. the hindbrain, then midbrain, then forebrain. This is logical given that the need to eat and breathe is more important for survival than the ability to read and understand a newspaper. Neural tube defects Failure of the neural tube to form correctly leads to a variety of congenital defects: Fig. 28.1 Embryology of central nervous system. A. The nervous system develops from the neural plate, and the area of ectoderm. Dorsal view at 18 days. B. Transverse section showing the neural plate and early development of the neural groove and neural folds. C. Neural folds are open at both ends. Dorsal view at 22 days. D. Transverse section showing formation of the neural tube. The neural tube differentiates into the CNS, i.e. the brain and spinal cord. The neural crest forms most of the peripheral and autonomic nervous system. E. The three primary vesicles, the forebrain, midbrain and hindbrain and the walls and cavities formed from them. (Adapted from Fig. 16.1 in Moore KL, Persaud TVN, Torchia MG. Before we are born, 8th edition, Saunders 2013, with permission.) A B C D E Oropharyngeal membrane Neural plate Neural plate Primitive knot Notochordal process Level of section B Level of section D Primitive streak Cloacal membrane Neural fold Rostral neuropore Caudal neuropore Somites Neural groove Neural fold Neural groove Amnion Wall of umbilical vesicle Intraembryonic mesoderm Notochordal plate Surface ectoderm Notochord Neural tube Neural crest Dorsal aorta Umbilical vesicle Wall Cavity Forebrain (prosencephalon) Midbrain (mesencephalon) Hindbrain (rhombencephalon) Walls Cavities Cerebral hemispheres Lateral ventricles Thalami, etc. Third ventricle Midbrain Aqueduct Pons Upper part of fourth ventricle Medulla Cerebellum Lower part of fourth ventricle
545 CHAPTER TWENTY-EIGHT Spina bifida comprises a spectrum of disorders: • Spina bifida occulta – a defect in development of the vertebral arch, which is covered by intact skin and does not involve the spinal cord. The lumbar spine is the most frequent location and occurs in up to 10% of healthy people. In asymptomatic people, it is usually identified on incidental X-rays. If there is an overlying patch of hair or naevus or other skin abnormality, tethering of the spinal cord may occur during childhood. An ultrasound or MRI scan of the spine should be performed and a neurosurgical opinion obtained. • Spina bifida cystica – more severe neural tube defect in which spinal cord tissue and/or meninges protrude through the skin. Most cases involve the lumbar spine. Includes meningocele and myelomeningocele: – Meningocele – the fluid filled meninges, but not the spinal cord, protrude through the defect – Myelomeningocele – neural tissue is involved and protrudes. These lesions are often associated with downward displacement of the cerebellar tonsils through the foramen magnum, called the Chiari malformation (also known as Arnold–Chiari malformation), which may cause non-communicating hydrocephalus. Cell differentiation Once the basic three-region structure of the brain is completed, it must be populated with active brain cells. Precursors of brain cells are the pluripotent neural stem cells. These differentiate into neurons, astrocytes, and oligodendrocyte lineages: • Neurons: – comprise a cell body (soma), dendrites and a single axon that terminates in one or more synapses – are electrically excitable – rely on metabolically active ion channel pumps to maintain voltage gradients – action potentials arise via voltage-gated ion channels • Astrocytes: – dominant cell line in the brain and spinal cord – star shape – support blood–brain barrier – neurotransmitter recycling – modulate synaptic signaling – maintain extracellular ion balance – provide nutrients to neural cells – brain repair and scarring – can transform into neurons Spina bifida A combination of genetic and environmental factors contribute to spina bifida. After having an affected infant, the risk of a second is 3–5%, and after two affected children have been born to a mother, the risk to the third is approximately 5–10%. Drugs are known to have an effect. There is a 10–20 times increased risk in mothers taking valproate. The biological availability of folate to the fetus appears pivotal to the observed risk. The prevalence is reduced by maternal folic acid supplementation before and during early pregnancy, although in most people, the maximum effect is probably reached at small daily doses. Cereal grain products are fortified with folic acid in the United States but not in the UK or most countries in Europe. There has been a marked decline in the prevalence of neural tube defects in the UK, due to improved maternal nutrition, folic acid supplementation and antenatal screening, mainly with ultrasound, and the option of termination of pregnancy. In the UK, 1.2 per 1000 pregnancies are affected; the birth prevalence is 0.2 per 1000 live births. Genetic polymorphisms in genes that are involved in folate metabolism such as methylenetetrahydrofolate reductase (MHTFR) show strong associations with the risk of spina bifida and anencephaly. Polymorphisms (variations) in this gene are common in the general population, though, and most do not lead to abnormalities in the fetus. The exact mechanism of risk is not known but is likely to be related to the folate–homocysteine–methionine pathway. Question 28.1 Spinal swelling A baby boy is born after a concealed pregnancy, and is estimated to be at term. A 6 cm thin-walled, fluid-filled swelling is noted at the base of the spine. He appears healthy but has weakness in his lower limbs. What is the approximate risk of recurrence in mother’s next pregnancy? A. 0.5% B. 3% C. 7% D. 10% E. 15% Answer 28.1 B. 3%. See below for discussion.
28 546Neurology Cell migration Neural cells migrate to their correct locations along specially created ‘highways’ of glial cells, known as radial glia. Each layer of brain cells is laid down, and then forms a scaffold upon which the next layer can be built on top – in other words, the brain develops ‘inside out’. When this process is complete, the cortex has six distinct layers of organization. This amazingly complex system can, and does, go wrong (Table 28.1). Neuronal migration disorders can be visible on MRI scan (for example, lissencephaly – lack of development of brain folds) and are associated with a wide spectrum of clinical difficulties, including some forms of epilepsy. Synaptogenesis Synaptogenesis is the creation of functional signalling mechanisms between neurons. It ‘explodes’ at around 20 weeks’ gestation, and continues for the first couple of years after birth. In the central nervous system, a projection from a dendrite (a filopodium) makes contact with an axon. This contact triggers the axon to recruit synaptic vesicles and active proteins to the area, which now forms the presynaptic membrane. Neurotransmitter receptors gather in the membrane of the dendrite, i.e. the postsynaptic membrane. Both cells • Oligodendrocytes: – support and insulate the axon – permit saltatory conduction of nerve impulses (propagation of action potentials along myelinated axons from one node of Ranvier to the next) – increase conduction velocity along neurons – decrease metabolic workload of neurons Table 28.1 Summary of embryology of the central nervous system and associated disorders Gestation CNS development Associated disorders Weeks 3–4 Primary neurulation Spina bifida Anencephaly Months 2–3 Prosencephalic development Holoprosencephaly Agenesis of the corpus callosum Septo-optic dysplasia Months 3–4 Neuronal proliferation Microencephaly Macroencephaly/cerebral gigantism Months 3–5 Neuronal migration Schizencephaly (grey matter lined clefts extending inwards to the ventricles) Lissencephaly (absence of normal sulci and gyri) Polymicrogyria Heterotopias Focal cortical dysplasia Month 5 onwards Neuronal organization Learning disability Link to epilepsy and autism Birth through childhood Myelination Range of disorders including adrenoleukodystrophy Question 28.2 The timing of CNS development A baby is born at term following a concealed pregnancy to a 17-year-old mother with depression. During pregnancy, she was treated with Drug A for the first 4 weeks before changing to Drug B for the next 8 weeks, Drug C (alone) for months 3–6 and Drug D (alone) during the final trimester. She took Drug E in the year prior to conception but coincidentally stopped it in the week before she fell pregnant. Drug E Drug B Drug C Drug D Drug A First trimester Second trimester Third trimester Conception Birth For the following defects, select the MOST likely drug (Drugs A–E) with the observed abnormality: 1. Agenesis of the corpus callosum 2. Anencephaly 3. Lissencephaly Answer 28.2 1. Drug B (taken between 2 and 3 months). 2. Drug A (although Drug E might also be implicated here, Drug A is still more likely). 3. Drug C. See Table 28.1 for rationale.
547 CHAPTER TWENTY-EIGHT channels open. Positive ions enter the target neuron and depolarize the membrane, decreasing the difference in voltage between the inside and outside of the neuron. When this reaches the threshold potential (−55mV), Na+ channels open. Na+ rapidly enters the cell and the neuron completely depolarizes to a positive membrane potential of about +40mV. The action potential travels down the neuron as more Na+ channels open. Once an action potential has been generated, the Na+ ion channels close, and the cell is refractory to any further stimulus. Voltage-gated K+ channels open, and K+ ions leave the cell. This takes the membrane potential down below the original −70mV (hyperpolarization), resetting the Na+ channels ready for the next signal. The cell then ‘resets’ itself by once again pumping out Na+ in exchange for K+ and returns to a voltage differential of around −70mV. Ion channels are of clinical relevance in certain diseases and also as targets for therapeutic drugs. As well as the voltage-gated ion channels mentioned above, channels may be ligand-gated (activated by ligand binding), mechanosensitive (activated by stretch and mechanical forces), cyclic-nucleotidegated (activated by substances such as cAMP), lightgated (in the eye), and temperature-gated. Conditions known to be caused by ion channel malfunction include: many of the epilepsies, including generalized epilepsy with febrile seizures plus (GEFS+), Dravet syndrome (SCN1A mutation), familial hemiplegic migraine, and some of the ataxias. Drugs affecting ion channels include lidocaine (a local anaesthetic, blocks sodium channels), phenytoin and carbamazepine (antiepileptics, block sodium channels), ondansetron (an antiemetic, ligand binding to 5-HT3 receptors), lamotrigine (an antiepileptic, ligand-gated channels) and gabapentin (an antiepileptic, ligand binding to inhibit calcium channels). Cell growth and apoptosis Neuronal cell growth depends on many things, including the hormone nerve growth factor (NGF). This protein, and other cell-signalling protein known collectively as ‘neurotrophins’, are responsible for the growth, maintenance, and survival of neurons. NGF has a role in myelin formation and also inhibits cell apoptosis. The precursor of NGF (pro-NGF) is also important in its own right. Although not biologically active in itself, in combination with tumour necrosis factor type receptors it can either promote cell growth or conversely promote apoptosis. Cell apoptosis or ‘programmed cell death’ is an essential part of the reorganization of the brain that occurs during growth and development of the child now express a protein called N-cadherin, which stabilizes the synapse. This now matures into the familiar arrangement, with a presynaptic (axonal) and postsynaptic (dendritic) membrane, separated by the synaptic cleft. A myelin sheath forms around thestructures, and the synapse is complete. Myelination Neuronal cells consist of the cell body, branching dendrites that receive information from surrounding cells, and long axons to carry information in the form of electrical impulses. Many axons are covered in myelin, produced by glial cells. Myelin can be thought of as being like insulation on an electrical wire; it increases the speed and accuracy of the transmission of information in the form of action potentials that propagate along neurons. The process of myelination begins in utero, and between 36 weeks and term myelin increases from 1% to 5% of total brain volume. Normal myelination progresses in a set sequence, beginning around birth and infancy with the basic senses of sight, smell, taste, touch and sound. From then onwards into adulthood, myelination continues in areas of the brain linked to thought, emotion, executive planning, and conceptualization. Just like other aspects of neuronal maturation, a child’s experiences and learning influence the rate and growth of myelination. Ion channels and the action potential As nerve cells rely on voltage gradients to produce an action potential, the nerve cell must maintain a membrane potential by modulating the concentration of intracellular ions. In a resting state, a nerve cell has high intracellular potassium and low intracellular sodium. The voltage gradient is maintained by two mechanisms: 1. The cell membrane has many potassium channels, which allow passive movement of potassium out of the cell along the concentration gradient, but few sodium channels to allow sodium to diffuse into the cell. 2. An adenosine triphosphate-dependent (ATP) pump actively transports three positively charged sodium ions (Na+ ) out of the cell in exchange for two positively charged potassium ions (K+ ). This energy-dependent process leads to a voltage differential across the cell membrane of around −70mV, and a high potassium concentration inside the cell relative to outside. In order to depolarize, ion channels in the cell membrane must activate and trigger ion redistribution. When neurotransmitter molecules bind to receptors located on a neuron’s dendrites, voltage-gated ion
28 548Neurology Neuronal plasticity When first developing, the human brain divides into two distinct hemispheres, linked via a large bundle of fibres called the corpus callosum. Initially, the two halves are mirror images of each other, and share functions. Over time, functions begin to lateralize to one side or the other, and ‘handedness’ develops. The ability to change and strengthen neuronal pathways is known as ‘plasticity’. Put simply, pathways that are used grow stronger, and those that are not used will diminish and may disappear altogether. Figure 28.2 illustrates lateralization relating to hand function. Initially, the right hand is supplied by motor fibres from both hemispheres. Around the time of birth and shortly afterwards, contralateral fibres from the left cerebral hemisphere begin to dominate, and the ipsilateral pathway from the right hemisphere diminishes. This explains why brain damage at different stages of development produces different clinical outcomes. If, for example, an infant suffers a left hemispheric stroke early in utero, the right hand is still (see Neuronal plasticity, below). NGF is thought to have a role in many neurological conditions, including psychiatric and neurodevelopmental disorders, and to be a potential therapeutic agent for neurodegenerative conditions and multiple sclerosis. Craniocaudal and centrifugal development Once the basic structure of the nervous system is formed, the development of skills and abilities follows a craniocaudal (head to tail) and centrifugal (central to peripheral) sequence. This can be clearly seen in the way infants gain motor skills. First to emerge are essential skills, such as coordinated suck and swallow, then head control, then shoulder and upper trunk stability to reach for toys, followed by hip stability and sitting, then lower limb control for standing and walking. Peripheral skills, such as fine motor coordination to use a tripod grasp and draw pictures, come later than the central skills, such as stabilizing the shoulders to allow the arm to push their dinner plate off the high chair. Fig. 28.2 Schematic representation of the differing effects of injury (orange circle) to primary motor cortex at different ages. The final, adult architecture (far right in each row) differs in each case because of interaction between the injury and developmental milestones; shown here is the physiological regression and loss of ipsilateral corticospinal projections (blue) that occurs in normal early postnatal life due to competitive inhibition from the contralateral corticospinal tract (green). (From Forsyth R. Back to the future: rehabilitation of children after brain injury. Arch Dis Child 2010;95:554–9, with permission.© BMJ.) Intact Prenatal injury Early postnatal injury Late injury Prenatal Early postnatal Late postnatal Adult
549 CHAPTER TWENTY-EIGHT Furthermore, an older person has the benefit of experience and has pre-established pathways that can augment both learning and speed of cognition. supplied by both sides of the brain at that time. The left side simply takes over control of both hands, and the infant retains bimanual function. If the same injury occurs in the postnatal period, there may still be some capacity for ipsilateral pathways to develop. The right hand will be weak, but may have some function. In a young adult, the same stroke will eliminate the contralateral innervation of the right hand, and with no remaining ipsilateral fibres to utilize, function is lost. Fortunately, even adults may still have some residual ipsilateral fibres; people who have less hand preference or who have bilateral innervation show greater scope for recovery of function. When developing, neurons have a wealth of synaptic connections. By age 3, the brain has around 1000 trillion synapses. Synaptic connectivity is maximal in early childhood and it is at this time that the brain is most receptive to the accumulation of new skills and knowledge, and maximum learning is possible. Neuroplasticity is of utmost importance in visual development. As demonstrated by the experiments of Hubel and Wiesel (who famously studied visual cortex firing in cats): without binocular visual stimulation, anatomical abnormalities develop in the ocular dominance columns of the lateral geniculate nucleus and visual cortex. Visual deprivation in the first 3 months of life in infants (the critical period of visual neuroplasticity) may result in permanent visual impairment (amblyopia). The neonatal (performed within 72 hours after birth) and GP (at 6–8 weeks) eye screening checks should detect and enable the management of ocular abnormalities, such as cataract, within this period. The higher visual pathway continues to develop and differentiate over the first eight years of life (the sensitive period of visual neuroplasticity); visual deprivation, strabismus (squint) or uncorrected refractive error during this period will result in amblyopia. Young children have more capacity for plasticity than adults, and can make remarkable recovery from significant brain injuries. However, the true extent of the damage may not become known for many years, when ongoing maturation ‘unmasks’ a neurological deficit. An example is traumatic brain injury, when the child may seem to make a full recovery in terms of returning to previous level of function, only to show significant cognitive difficulties when reaching adolescence. Similarly, it is easier to regain a function lost as a teenager than develop it from scratch as a toddler; for example, a concert pianist who has had a stroke will find it easier to learn to play again than a similar stroke sufferer who has never touched a piano. Although synaptic pruning means that connections are lost throughout puberty and is complete soon after the end of puberty, learning and therefore a degree of plasticity is possible throughout life and into old age. Question 28.4 The blood–brain barrier Which of the following substances is least able to cross the blood–brain barrier? Select ONE answer only. A. Ammonia B. Amoxicillin C. Carbon dioxide D. Glutamine E. Unconjugated bilirubin Question 28.3 Neurological development Which of the following statements are true (T) and which are false (F)? A. Anencephaly results from failure of closure of the anterior neuropore B. Myelin develops from birth C. Neurones migrate along myelinated bundles to their eventual position in the brain D. The maximum number of synaptic connections are present at term E. The retina originates from the forebrain (prosencephalon) Answer 28.3 A. True; B. False; C. False; D. False; E. False. Whilst brain development is complex, it is well studied and much is known. Anencephaly results from failure of closure of the anterior neuropore but all our other answers are false. Neural cells migrate to their correct location along specially created ‘highways’ of glial cells, known as radial glia. The retina originates from the diencephalon, along with the thalamus and hypothalamus. The forebrain (prosencephalon) gives rise to the cerebral hemispheres and basal nuclei. Myelination begins in utero, and between 36 weeks and term increases from 1% to 5% of total brain volume. Synaptic connectivity is maximal in early childhood rather than at birth. It is at this stage that the brain is most receptive to the accumulation of new skills and knowledge. As the brain matures, unused synaptic connections are lost via the process of ‘synaptic pruning’.
28 550Neurology The blood–brain barrier The brain and spinal cord are uniquely protected by a physical barrier that isolates them from biochemical changes in the rest of the body. This shield is known as the blood–brain barrier (BBB). Not to be confused with the interface between cerebrospinal fluid (CSF) and the brain, the BBB lies along the individual capillaries that feed deep into the brain tissue itself. It is formed by closely sealed endothelial cells in cooperation with brain astrocytes, and regulates and adjusts transfer of nutrients, ions, proteins, and components of the immune system. It achieves this by highly selective permeability and use of specialized transporters. Glutamine differs from other amino acids in that it passes across the BBB through such a mechanism. Facilitative carriers for glutamine and glutamate at the luminal membrane may provide a mechanism for removing nitrogen and nitrogen-rich amino acids from brain. Some molecules are able to freely cross the BBB: water, gases, and also some lipid-soluble substances. Increased BBB permeability for ammonia is considered to be an integral part of the pathophysiology of hepatic encephalopathy. This permeability to lipidsoluble agents is clinically important. In the jaundiced neonate, unconjugated bilirubin is highly lipidsoluble. As such, it passes readily across the BBB and into the brain, leading to neurological damage and kernicterus. Exposure to light of a specific range of wavelengths converts the bilirubin to a water-soluble state, by converting insoluble bilirubin into watersoluble structural isomers. It also generates bilirubin molecules in an excited state, and these react with oxygen to produce colourless oxidation products. Once the molecules are water-soluble they can no longer cross the BBB, and thus no longer damage the brain. Lipid solubility is also clinically relevant when prescribing. Agents that are lipid soluble are more likely to cross the BBB and alter CNS function. This may be desirable, for example when giving a general anaesthetic, but may lead to unwanted effects of sedation or brain disturbance with other drugs. The BBB is susceptible to damage, in particular from inflammation in conditions such as meningitis. This is mediated by pro-inflammatory cytokines, and leads to opening up of spaces between endothelial cells and loss of the tight regulation of transport across the BBB. This can be advantageous (for example, allowing white blood cells and immune components to enter the brain), but can also lead to chemical imbalances and cell damage. In meningitis, however, we capitalize on the fact that the BBB is no longer preventing entry of chemicals into the brain, as it permits a far greater penetration of prescribed antibiotics (including the water-soluble amoxicillin) than would ordinarily be possible. Anatomy and physiology of the nervous system Knowledge of the structure and physiology of the nervous system allows understanding of how damage to different areas may present. The general structure of the brain is shown in the sagittal view in Figure 28.3. At the spinal level, 31 pairs of mixed motor and sensory nerve bundles emerge (Fig. 28.4): • Cervical (C): 8 pairs • Thoracic (T): 12 pairs • Lumbar (L): 5 pairs • Sacral (S): 5 pairs • Coccygeal: 1 pair These nerves further combine to produce nervous plexi: • Cervical plexus (C1–C4): muscles of the neck, shoulder and skin, phrenic nerve (diaphragm) • Brachial plexus (C5–C8 + T1): muscles from the base of the neck to the fingertips and skin • Lumbar plexus (L1–L3 + part of L4): skin and muscles of the lower abdomen, thighs and groin • Sacral plexus (L4–L5 + S1–S4): muscles and skin of the pelvic area and legs • Coccygeal plexus: muscles and skin of pelvic area and sphincters Cerebrospinal fluid Distinct from the BBB, the CSF–brain barrier relates to the extracerebral fluid that is found within the ventricles and around the brain. It has two main categories of function: physical and biochemical. From a physical standpoint, it acts to cushion and protect the brain from shear forces and impact, and plays a role in regulating intracerebral blood pressure and thus prevents ischaemia. Biochemically it serves to remove waste and toxins from the CNS, and helps regulate levels of hormones and neurologically active substances. CSF is produced by a type of glial cell called an ependymal cell. It is chiefly produced in the choroid plexi in the lateral ventricles of the brain, and exits through the intraventricular foramen of Munro, into the third ventricle, through the aqueduct of Sylvius into the fourth ventricle, then down the spinal cord and over the cerebral hemispheres. It is reabsorbed Answer 28.4 B. Amoxicillin. See below for discussion.
551 CHAPTER TWENTY-EIGHT insertion of a one-way valved ventriculoperitoneal shunt, which forms a direct drainage route for CSF from the cranial vault to the low pressure of the peritoneal cavity. Clinical features of hydrocephalus depend on the site of the obstruction and also on the capacity of the cranial vault to expand if the sutures are not yet fused. Symptoms of hydrocephalus may be acute (vomiting, irritability, headache, change in consciousness) or chronic (visual disturbance, ‘sunsetting eyes’, early morning vomiting, pressure headache, deterioration in school performance). Untreated, acute hydrocephalus (for example, in a child with a blocked ventriculoperitoneal shunt) can lead to brainstem herniation and death. It is a neurosurgical emergency. CSF analysis is of value in many disorders, the most common of which is meningitis (Table 28.2). It is also of value in diagnosis and management of metabolic disorders, leukaemias, neurodegenerative conditions and autoimmune disorders. Meningitis CSF microscopy (see Table 28.2) may initially be normal in meningitis, so clinical impression is of prime importance. It is unusual to find neutrophils in CSF beyond the neonatal period, so this should raise the possibility of bacterial infection. CSF white cell into the circulation via arachnoid villi. CSF is produced at a rate of around 30 mL/hour. The volume after the age of 2 years is around 150 mL, with about 35 mL in the ventricular system. Abnormalities of the CSF circulation may result in hydrocephalus: • Communicating hydrocephalus – no obstruction between ventricles and subarachnoid space. Caused by: – Excessive CSF production (rare, e.g. choroid plexus tumour) – Impaired CSF resorption (e.g. blockage of arachnoid granulations by debris after meningitis or haemorrhage) • Non-communicating hydrocephalus – physical obstruction between ventricles and subarachnoid space. Caused by: – Congenital malformation (e.g. aqueduct stenosis, Arnold–Chiari malformation) – Acquired obstruction (e.g. brain tumour) Idiopathic intracranial hypertension is a special case where the CSF is elevated in the absence of hydrocephalus or intracranial mass lesion, and is described later in this chapter. Treatment of hydrocephalus depends on the cause. It may be via resection of an intracranial obstruction, placing of a stent in a stenosed aqueduct, or by removal of excess CSF. In many children this requires Fig. 28.3 General anatomy of the brain in the sagittal view. (From Naish J, et al. Medical sciences, 2e. Saunders 2014, with permission.) Superior cistern Cerebellum Fourth ventricle Cerebello-medullary cistern Spinal cord (cut) Lumbar cistern Medulla Pontine cistern Interpeduncular cistern Optic chiasma cistern Pituitary gland Lateral ventricles Pons Foramen of Magendie (and Luschka) Choroid plexus of fourth ventricle Third ventricle Choroid plexus of lateral ventricles Arachnoid Bulk flow of CSF into venous sinuses via arachnoid villi Subarachnoid space
28 552Neurology difficult to distinguish between viral and bacterial meningitis. Antibiotics do not significantly change the CSF cell count or biochemistry in samples taken within 24 hours of the initial dose. Meningococcal PCR testing on CSF samples is particularly useful in patients with a clinical picture consistent with meningococcal meningitis but who have received prior antibiotics. CNS inflammatory diseases CSF IgG may be raised in patients with central nervous inflammatory diseases (e.g. multiple sclerosis, subacute sclerosing panencephalitis). The most commonly used diagnostic tests for multiple sclerosis are a raised CSF index (the ratio of CSF IgG to CSF albumin compared to the serum IgG to serum albumin ratio) and oligoclonal band detection. CSF neurotransmitter disorders The monoamine neurotransmitter disorders are caused by defects in the synthesis, degradation, or transport of dopamine, noradrenaline (norepinephrine), adrenaline (epinephrine), and serotonin. They are implicated in a varied group of conditions including mitochondrial disorders, Rett’s syndrome and leukodystrophies. They usually present with abnormal neurological features (encephalopathy, epilepsy, and pyramidal and extrapyramidal motor disorders). Their diagnosis may include analysis of neurotransmitters in the CSF, which involves a precise protocol for collection and rapid freezing of samples. Specialized laboratory analysis is required. History and examination History History is the cornerstone of neurological diagnosis. Particular attention should be paid to the age at count and protein level are higher at birth than in later infancy. In the first week, 90% of well babies have a white cell count less than 20, and a protein level <1.0 g/L. Although lymphocytes are more characteristic, neutrophils may predominate in viral meningitis even after the first 24 hours, which may make it Fig. 28.4 Spinal nerves. (From Naish J, et al. Medical sciences, 2e. Saunders 2014, with permission.) Cervical nerves C1–C8 Thoracic nerves T1–T12 Lumbar nerves L1–L5 Sacral nerves S1–S5 Coccygeal nerve Co1 Cervical plexus Brachial plexus Lumbar plexus Sacral plexus Intercostal nerves Lumbar enlargement Cervical enlargement Cauda equina Table 28.2 CSF analysis values in diagnosis of meningitis* White cell count Biochemistry Neutrophils (× 106 /L) Lymphocytes (× 106 /L) Protein (g/L) Glucose (CSF:blood ratio) Normal (>1 month of age) 0 ≤5 <0.4 ≥0.6 (or ≥2.5 mmol/L) Normal term neonate <5 <20 <1.0 ≥0.6 (or ≥2.5 mmol/L) Bacterial meningitis 100–10,000 (but may be normal) Usually <100 >1.0 (but may be normal) <0.4 (but may be normal) Viral meningitis Usually <100 10–1000 (but may be normal) 0.4–1 (but may be normal) Usually normal TB meningitis Usually <100 50–1000 (but may be normal) 1–5 (but may be normal) <0.3 (but may be normal) *Although helpful, treatment should not be delayed if it is not possible or safe to perform a lumbar puncture.
553 CHAPTER TWENTY-EIGHT onset, developmental history including age of milestones, any loss of skills or deterioration of school performance, detailed family history, and the impact of the problem on family life and functioning. Typically, presentation falls into one of the following categories: • Developmental delay – single domain or multidomain • Developmental regression – e.g. Rett’s syndrome • Weakness – e.g. muscular dystrophy • Abnormal movements – e.g. ataxia • Change in level of consciousness – e.g. encephalopathy • Paroxysmal events – e.g. epilepsy • Headache – e.g. migraine • Traumatic injury – e.g. head injury Examination A full neurological examination is a skilled and lengthy procedure, and is not required unless a neurological disorder is suspected. In those who do have a neurological symptom, your history will allow you to focus your attention on a specific part of the neurological system, with a more generalized exam of the rest of the child. Detailed neurological examination can be tricky in children, as you need a cooperative patient! However, it is usually possible to gain a good assessment of ability through a combination of observation of play and use of ‘games’ to encourage the child to do what you need. For example: a child may not cooperate with a ‘finger–nose pointing’ test, but is likely to join in with a ‘sweetie–mouth’ test if you hold out a pretend sweet for them to grasp. A child may not want to show you their gait or to demonstrate shoulder abduction for you, but is likely to join in a ‘race’ or to ‘flap your elbows like a chicken’. Examining the neonate and infant (Box 28.1) With very young babies, starting with the baby lying supine on their back, you move them through sitting, to prone, and then to lying on their back again. At all stages, assess the symmetry of their limb movements and reactions. Always take any concerns from parents seriously. Examining the older child In older children, it is helpful to consider examination of each area of the nervous system individually: the cranial nerves (Fig. 28.5), cerebellum, upper limb, lower limb, and gait (Boxes 28.2–28.6). Box 28.1 Examining the infant Supine: • Assess general movements, posture, and alertness • Check for birthmarks (neurocutaneous markers) • Measure and plot head circumference • Assess fontanelles (hydrocephalus) and head shape (craniosynostosis) • Lift head, and if you wish to elicit the Moro reflex (usually unnecessary and unpopular with parents), allow head to fall back quickly supported in your hand by a short distance, assess reaction of arms • Individually assess tone, power, movement and reflexes in each limb (remember Babinski will be positive) • Observe response to sound • Assess pupil responses, reaction to light, object fixation and red reflexes/fundoscopy • Take both hands (grasp reflex) • Pull baby by hands to sitting (head lag) Sitting: • Degree of back rounding (tone) • Ability to self-support (tone and posture) • Grasp gently around the chest below each axilla and lift Standing: • ‘Slipping through hands’? (hypotonia) • Rigid legs (hypertonia) • Stepping and walking reflexes (neonatal reflexes) • Lay infant prone across your hand/forearm Prone suspension: • Degree of drape/ability to lift head and legs (tone) • Examine spine for abnormalities or defects • Stroke side of spine to see infant curl towards stimulus (Galant reflex) • Lay infant back down on the bed, prone (head raise, rolling, attempt to crawl) • Turn infant onto their back again, redress infant
28 554Neurology Fig. 28.5 The cranial nerves. (© 2016 Elsevier Inc. All rights reserved. www.netterimages.com) I Olfactory II Optic III Oculomotor Ciliary m., sphincter of pupil, and all external eye mm. except those bclow IV Trochlear Superior oblique m. VI Abducens Lateral rectus m. VII Facial Mm. of face XII Hypoglossal Tongue mm. XI Accessory Sternocleidomastoid, trapezius mm. X Vagus Motor–heart, lungs, palate, pharynx, larynx, trachea, bronchi, GI tract Sensory–heart, lungs, trachea, bronchi, larynx, pharynx, GI tract, external ear IX Glossopharyngeal Taste–posterior 1/3 of tongue Sensory–tonsil, pharynx, middle ear Motor–stylopharyngeus, upper pharyngeal mm., parotid gland VIII Vestibulocochlear Intermediate n. Motor–submandibular, sublingual, lacrimal glands; Taste–anterior 2/3 of tongue, sensory soft palate V Trigeminal Sensory–face sinuses, teeth Motor–mm. of mastication Ophthalmic Maxillary Mandibular Cochlear Vestibular Spinal n. fibres Efferent (motor) fibres Afferent (sensory) fibres
555 CHAPTER TWENTY-EIGHT Box 28.2 Cranial nerve examination 1. Olfactory nerve (smell) a. Often not tested and, if required, is assessed by asking the patient if they have noted any change in their sense of smell if test done on each nostril in turn 2. Optic nerve a. Visual acuity (by observation or with Snellen chart) b. Visual fields (see Chapter 30, Ophthalmology, for more detail) c. Pupillary reflexes – assess for direct and consensual responses to light, subsequently testing for accommodation. d. Fundoscopy 3. Oculomotor nerve – examine as below 4. Trochlear nerve – examine as below 5. Trigeminal nerve a. Motor component – supplies the muscles of mastication i. Ask the child to open their mouth, whilst you try to close it. It will deviate to the side of weakness ii. Clench teeth together and feel muscle mass of the masseters b. Sensory component – ophthalmic, maxillary and mandibular divisions i. Examine by testing each division comparing both sides ii. Do not assess the corneal reflex 6. Abducens a. Examine cranial nerves 3, 4 and 6 by asking the child to fix and follow on an object (lots of people use their finger although the light of a pen torch is often more successful) 7. Facial a. Motor component supplies the muscles of facial expression i. Ask the child to close their eyes tightly, open their eyes really wide, blow out their cheeks and show their teeth b. Sensory component supplies the anterior two thirds of the tongue 8. Vestibulocochlear a. You should be aware of how to perform a Rinne’s/Weber’s test. 9. Glossopharyngeal – examine as below a. Motor component – supplies stylopharyngeal muscle b. Sensory component supplies: i. Posterior third of tongue for taste ii. Tonsillar fossa and pharynx 10. Vagus nerve – examine 9 and 10 together by asking the child to open their mouth and say ‘ahh’ whilst looking at the palatal movement. Gag reflex should be considered but not performed. a. Motor component supplies pharynx and larynx b. Sensory component supplies larynx alone 11. Accessory nerve – ask the child to shrug their shoulders, turn their head to the right and place a hand on the left side of their face and ask to push against your hand, test in the opposite direction. 12. Hypoglossal nerve a. Ask the child to stick their tongue out and inspect for fasciculation and deviation (tongue deviates to the side of the lesion) Box 28.3 Cerebellar examination 1. General observation looking for any abnormal movements, abnormal speech and telangiectasias on the conjunctiva 2. Assess speech by asking them some questions, all the while taking note of stuttering or dysarthria 3. Eyes a. Look for nystagmus b. Assess eye movements, especially on moving the eyes to the extremes of horizontal gaze 4. Upper limbs – essentially you are performing a selected upper limb neurological examination a. Assess for tone – hypotonia is usual in cerebellar disorders b. Coordination i. Dysmetria using finger–nose test (see earlier) ii. Dysdiadochokinesia iii. Piano-playing – ask the child to pretend to play an imaginary keyboard c. Reflexes – hyporeflexia is usual 5. Lower limbs a. Assess for tone – hypotonia b. Coordination i. Heel–shin test ii. Toe-tapping – ask the child to tap your hand with the sole of the right foot as quickly as possible c. Reflexes 6. Gait – ask the child to walk normally and then heel–toe whilst looking for a broad-based gait
28 556Neurology Box 28.5 Lower limb examination It is always important to examine gait as part of a lower limb assessment, and this is described separately. 1. General observation – as mentioned above, take note of clues such as the child’s wheelchair, orthoses or splints, helmet or dysmorphic features 2. Ask the child and parent to undress to their underpants only 3. Assess the child’s gait 4. Inspection – with the child lying on the bed, inspect the child’s legs and spine. Note: a. Posture b. Contractures c. Muscle bulk, wasting and (pseudo) hypertrophy d. Involuntary muscles and fasciculations e. Scars 5. Assess tone: a. Roll the legs from side to side taking note of movement of the foot at the ankle b. Place the palmar surfaces of both one’s hands underneath the child’s thighs and lift the thighs up slightly off the bed and then let them fall c. Flex and extend at the hip, knee and ankle joints d. Clonus – test both ankles by rapidly dorsiflexing the foot – more than 3 beats of ankle clonus is abnormal 6. Power a. Hip flexion (L1 and L2) and extension (L5 and S1) b. Knee flexion (L3 and L4) and extension (S1) c. Ankle dorsiflexion (L4) and ankle plantar flexion (S1 and S2) Box 28.4 Upper limb examination 1. General observation – take note of clues such as the child’s wheelchair, orthoses or splints, helmet or dysmorphic features 2. Ask the child and parent to expose the chest and arms fully 3. Inspection – with the child standing inspect for: a. Posture b. Contractures c. Muscle bulk, wasting and (pseudo) hypertrophy d. Involuntary muscles and fasciculations e. Scars f. Limb shortening 4. Tone – whilst holding on to their hand passively, move their arm in unexpected, irregular movements 5. Power – compare both sides a. Shoulder abduction (C5) and shoulder adduction (C7, 8) b. Elbow flexion (C5, 6) and extension (C7, 8) c. Hands: i. Squeeze my fingers ii. Spread your fingers out wide 6. Reflexes – compare both sides a. Biceps – C5, 6 b. Triceps – C7, 8 c. Brachioradialis – C5, 6 7. Coordination a. Finger–nose test (assess for dysmetria) – ask the child to touch their nose with their index finger, and then your finger. You must move your finger to different sites at different arm lengths. b. Hand-tapping (assess for dysdiadochokinesia) – get them to tap one hand on the back of another and to alternate tapping between the palm and back of the hand. 8. Sensation may be difficult to perform in children, but it is important to have a rough idea about dermatomes in case you need to assess a. C4 – tip of shoulder b. C5 – lateral surface of upper arm c. C6 – lateral surface of forearm d. C7 – radial surface of middle finger e. C8 – ulnar border of hand f. T1 – medial surface of forearm g. T2 – medial surface of upper arm 9. Function is assessed by asking the child to draw/write/pick up objects
557 CHAPTER TWENTY-EIGHT Box 28.5 Lower limb examination—cont’d 7. Reflexes – compare both sides a. Knee – L3, L4 b. Ankle – S1, S2 c. Plantar reflex – an abnormal reflex (i.e. positive Babinski reflex) is for the great toe to be up-going and therefore the toes to be splayed when stroking the lateral aspect of the sole of the foot and medially across the ball of the foot. (Positive Babinski is a normal finding in infants) 8. Coordination – test both sides by asking the child to run the heel of one foot down the shin of the other (heel–shin test). This is often best demonstrated to them using their foot rather than explaining what you want them to do verbally. 9. Sensation can be difficult to test in children but one needs to know the dermatomes. Test light touch, vibration and joint proprioception (pain sensation is not usually expected to be tested) a. L1 – upper outer thigh b. L2 – middle anterior thigh c. L3 – anterior knee d. L4 – medial calf e. L5 – lateral calf f. S1 – sole of the foot g. S2 – strip up posterior and thigh Box 28.6 Gait examination 1. General observation – again, take note of clues such as the child’s wheelchair, orthoses or splints, helmet or dysmorphic features 2. Ask the child and parent to undress the child to their underpants only 3. Inspection – with the child standing inspect for: a. Posture b. Contractures c. Muscle bulk, wasting and (pseudo) hypertrophy d. Involuntary muscles and fasciculations e. Scars f. Limb shortening g. Spine for scars or lesions (overlying tuft of hair, for instance, which may suggest spina bifida) h. Examine their shoes for evidence of abnormal wear and tear 4. Ask the child to walk across the room a. Assess for the normal heel-strike/toe-off phases b. Foot position – varus/valgus c. Noise of the walk – slapping gait associated with foot drop d. Limp e. Arm position and swing f. Abnormal movements 5. Ask the child to walk heel–toe – assessing cerebellar pathways 6. Ask the child to walk on the outsides of their feet (Fog’s test) – this exacerbates the signs of a subtle hemiplegia 7. Ask the child to run across the room – again exacerbating any signs of a subtle hemiplegia 8. Squat–stand – ask the child to stand from the squatting position – assess for proximal myopathy 9. Trendelenburg’s sign for proximal (hip) muscle weakness a. Ask the child to stand in front of you but facing away b. Get them to lift one foot off the ground i. Normal is for the pelvis to rise on the side of the lifted leg ii. The test is abnormal if the pelvis sags on the side of the lifted leg 10. Gower’s sign, by asking the child to stand from lying supine, which again assesses for proximal muscle weakness. Gower’s sign is when the child is unable to stand without turning prone and then bracing their hands on their knees or ‘walking’ their hands up their legs to get upright.
28 558Neurology Imaging the nervous system Cranial ultrasound This remains the modality of choice in neonates and infants due to the ease of scanning via the open anterior fontanelle. It is quick and non-invasive, but highly operator-dependent. Repeat scans over time allow evolution and progression of lesions to be monitored. It is particularly used for: • Intraventricular haemorrhage (IVH) and the ischaemic cysts of periventricular leukomalacia (PVL) • Ventricular dilatation • A range of cerebral malformations and other lesions, e.g. agenesis of the corpus callosum However, MRI is much better in detecting ischaemic lesions, e.g. hypoxic–ischaemic encephalopathy (HIE) or PVL and for detailed anatomy of cerebral malformations. Cranial computed tomography Cranial computed tomography (CT) is widely available and rapid, so continues to be used for: • Head trauma • If clinical condition is unstable • Intracranial calcification • Haemorrhage *Fluid attenuated inversion recovery (FLAIR) Question 28.5 Interpretation of MRI findings Which of the following patterns best describes the pattern of signal generated by cerebrospinal fluid on MRI? Select ONE answer only. A. T1 sequence bright, T2 sequence bright, FLAIR* sequence bright B. T1 sequence bright, T2 sequence dark, FLAIR sequence bright C. T1 sequence dark, T2 sequence bright, FLAIR sequence bright D. T1 sequence dark, T2 sequence bright, FLAIR sequence dark E. T1 sequence dark, T2 sequence dark, FLAIR sequence dark This technique can be enhanced with the use of contrast. Its use has largely been replaced by MRI, which does not require radiation and is usually more informative. Magnetic resonance imaging Magnetic resonance imaging (MRI) is the imaging technique of choice for most paediatric neurological disorders, e.g. for demyelination. No radiation is involved. Various different computerized sequences can be used to interrogate the signal to answer particular clinical questions (Table 28.3): • T1 sequence – CSF appears black, grey matter is grey and white matter white or paler grey. • T2 sequence – for assessing tissue fluid content, such as oedema. CSF is white. • FLAIR (fluid attenuated inversion recovery) – similar to T2, and is of value in interrogating lesions close to the ventricles. The signal from CSF is purposely reduced to allow clearer tissue imaging. Increased availability of MRI scanning in children has helped expand our knowledge about the pathological processes occurring in children with encephalopathy. Questions 28.6 and 28.7 illustrate this point. Questions 28.6 and 28.7 A child with acute neurological loss A nine-year-old girl presents with acute encephalopathy. In the preceding 24 hours, she was noted to have some fever and vomiting. Her parents also noticed that she was slightly confused the evening prior to her visual loss. Neurological examination revealed no focal deficit and there were no signs of meningism. Investigations revealed the following: Full blood count unremarkable, CRP 56 and ESR 32. Blood and urine cultures – negative CSF studies: • WCC <3, RCC <3, protein 0.16, glucose/lactate normal • Culture and virology – negative Table 28.3 Overview of MRI appearances T1 T2 FLAIR Solid mass Dark Bright Bright Fluid-filled cyst Dark Bright Dark Subacute blood Bright Bright Bright Acute and chronic blood Grey Dark Dark Fat Bright Dark Bright FLAIR, fluid attenuated inversion recovery. Answer 28.5 D. T1 sequence dark, T2 sequence bright, FLAIR sequence dark. See below for discussion.
559 CHAPTER TWENTY-EIGHT Question 28.6 What is the most likely diagnosis? Select ONE answer only. A. Acute disseminated encephalomyelitis (ADEM) B. Cerebral abscess C. Diffuse astrocytoma D. Multiple sclerosis E. Viral encephalitis Question 28.7 What is the most likely mechanism involved pathologically? Select ONE answer only. A. Direct spread of virus into white matter of brain B. Medium vessel vasculitis C. Perivenular infiltrates of lymphocytes and macrophages D. Small vessel vasculitis E. Tumour spread with secondary malignancy MRI brain. Multiple high signal white matter lesions on T2 weighted images around the grey–white matter junction. (From Swaiman KF, et al. Pediatric Neurology. Mosby 2006.) Answer 28.6 A. Acute disseminated encephalomyelitis (ADEM). Answer 28.7 C. Perivenular infiltrates of lymphocytes and macrophages. This case described in Question 28.6 highlights a classical presentation of acute disseminated encephalomyelitis (ADEM). The main differential diagnosis for this is multiple sclerosis (MS), although MS is rare in childhood. It is also essential to rule out meningo-encephalitis. It is important to try and differentiate between ADEM and MS based on history, clinical features and investigations. ADEM, an inflammatory demyelinating condition of childhood, commonly occurs after a viral or bacterial infection. Less commonly, it has been reported to occur post MMR vaccination. Most patients with ADEM have a degree of encephalopathy during the course of illness, which may progress to loss of consciousness. These children may also commonly have fever, vomiting and headache. Seizures may occur in up to a third of patients. All these features are rarely seen in MS and should alert a physician towards a diagnosis of ADEM. Viral encephalitis may have similar presenting features and therefore must be ruled out by performing a lumbar puncture. MS is a chronic inflammatory demyelinating disease of the CNS characterized by myelin loss and variable degrees of axonal degeneration and gliosis that correlate with progression of neurological disability. In MS, acute demyelination is part of a complex and incompletely understood immunological cascade in which there may be multiple episodes and a relapsing clinical course. CSF studies may show an increased protein and lymphocyte count in ADEM. However, these abnormalities are not exclusive to ADEM. Brain MRI is crucial in achieving a diagnosis. Both ADEM and MS show predominant white matter changes, which are disseminated. In ADEM, these lesions tend to occur in the deep white matter or around the grey–white matter junction, with periventricular sparing. In contrast, periventricular and corpus callosum white matter changes are highly suggestive of MS. Follow-up scans may also be useful, as in ADEM, new lesions should not occur. Pathologically, ADEM is characterized by perivenular infiltrates of lymphocytes, macrophages, and occasional plasma cells with oedematous and demyelinated adjacent white matter. Lesions are multifocal or diffuse and may be found in the optic nerves, spinal cord and brain. Characteristically, and in contrast to multiple sclerosis (MS), all lesions are of the same age, and axonal injury is minimal. Analysis of CSF for oligoclonal bands can be helpful in determining the risk of MS. Intrathecal synthesis of oligoclonal bands occurs in MS in 40–95% of cases, but only in 0–29% of children with ADEM. In either case of demyelination (ADEM or MS), the first line of management involves steroids,
28 560Neurology Recent scientific advances which have improved clinical practice – functional MRI Functional MRI allows for visualization of regional oxygen consumption and blood flow, and is used to examine brain activity. It allows creation of maps showing which parts of the brain are involved in a particular task, such as movement or speech. It is increasingly used in presurgical assessment of children being considered for epilepsy surgery and allows more accurate prediction of post-op functional outcome. This has led to the formation of a national paediatric epilepsy surgery service. It has also enhanced our understanding of other neurological conditions including autistic spectrum disorder. Question 28.8 An infant with abnormal movements Oliver, a 6-month-old, is referred by his GP to the paediatric day unit because of abnormal movements over the preceding two weeks. His parents initially noticed brief episodes lasting 2–3 seconds, where he would drop his head forward when he is sat up in association with forward tonic flexion of his upper limbs. They initially thought they were due to colic but became concerned when they increased in frequency, now happening 20–30 times daily. He has also been noted to be less responsive over the past few days. Oliver was born at term, with no neonatal problems and his development has been age-appropriate. On assessment, his systemic examination was unremarkable. No abnormal skin pigmentation was noted. He has had a cluster of episodes, which you suspect are infantile spasms. What would your next plan of action be? Select ONE answer only. A. Commence urgent treatment before any of the above B. Discharge with an urgent outpatient referral to paediatric neurology C. Discharge with outpatient paediatric follow-up D. Organize an urgent MRI of the brain within 48 hours E. Organize an urgent EEG within 48 hours Magnetic resonance angiography Magnetic resonance angiography (MRA) permits noninvasive assessment of the intracranial vascular system. It has found particular use in conditions such as intracranial arteriovenous malformations (e.g. Sturge– Weber syndrome) and blockages or stenosis. Magnetic resonance spectroscopy (MRS) This newer technique relies upon the individual resonance properties of certain molecules within brain tissue. Known patterns exist of chemicals such as lactate, choline, and creatine. It has many potential uses, as it can detect high levels of metabolites or other target molecules in conditions including HIE, brain injury, epilepsy, metabolic disorders, and multiple sclerosis. For example, a raised lactate signal suggests a metabolic disorder. Perhaps the most promising area of paediatric research at present is in tumours (choline signals are often elevated in tumour tissue). At present, MRS is restricted to larger research centres, and it is not yet widely available. Positron emission tomography This functional imaging technique is available in only a few centres. A labelled radioactive tracer is injected into the body, which gives off gamma rays that are detected by a scanner. The information gleaned depends on what molecule the tracer was attached to. If the target molecule is fluorodeoxyglucose (analogue of glucose), then the images will reflect regional glucose uptake and therefore the metabolic activity of the tissue. The main indication in paediatric neurology is to identify precise areas of seizure onset in focal epilepsy, thus identifying possible targets for epilepsy surgery. which have been shown to hasten recovery. It is common practice to give intravenous methylprednisolone for 3 days, followed by a tapering dose of oral steroids over a few weeks. However, as ADEM is a self-limiting condition, some cases start improving spontaneously before the introduction of steroids and may not require treatment. It is imperative that antivirals and antibiotics are continued until CSF culture/virology results are obtained. Answer 28.8 E. Organize an urgent EEG within 48 hours. If there are doubts regarding the diagnosis or if episodes are not witnessed by someone familiar with the condition, video recording by parents can be very useful. The main point to highlight is that any child with suspected infantile spasms should undergo an urgent EEG within 24–48 hours, where possible. Usually, an acute admission is necessary to facilitate this. An awake EEG may be normal in early stages and a period of sleep recording is therefore recommended for this reason. EEG patterns compatible with infantile spasms include hypsarrhythmia, modified hypsarrhythmia, and burst suppression amongst others. If the EEG is completely normal, including on follow-up, the likely diagnosis is benign myoclonus
561 CHAPTER TWENTY-EIGHT Neurophysiological investigations The electroencephalogram In routine electroencephalogram (EEG) monitoring, electrodes are positioned over the scalp. In invasive monitoring, electrodes are placed inside the skull and dura, but this is only used in quaternary epilepsy centres. Typically, 21 numbered electrodes are placed in defined positions in each of five locations (F – frontal, C – central, P – parietal, T – temporal, O Question 28.9 Neurophysiological investigation Following is a list of possible investigations: A. Electrocardiogram (ECG) B. Electromyography (EMG) C. Electroretinogram (ERG) D. Nerve conduction studies E. Standard electroencephalogram (EEG) without activation procedures F. Standard EEG with activation procedures (hyperventilation and photic stimulation) G. Sleep-deprived EEG H. Visual evoked potentials (VEP) From the list above (A–H), select the MOST appropriate investigation to identify the following disorders: 1. Myotonia 2. Optic neuritis 3. Peripheral nerve demyelination of early infancy, which can mimic infantile spams. This non-epileptic condition is harmless and resolves without any intervention. In many medical conditions, the principle of investigations prior to treatment applies. However, infantile spasms are an exclusion to this. Studies have shown that the longer it takes to control these spasms, the worse the outcome. Treatment should therefore be started within a few days of diagnosis, ideally within 24 hours, where possible. The International Collaborative Infantile Spasms Study (ICISS) should provide evidence about combined treatment with both hormonal treatment (steroids or ACTH) and vigabatrin compared to hormonal treatment alone. Roughly 40% of children with tuberous sclerosis complex (TSC) will have infantile spasms. It is important to look for TSC in all children presenting with infantile spasms as treatment differs – vigabatrin is the drug of choice for infantile spasms secondary to TSC. Answer 28.9 1. D. Nerve conduction studies 2. H. Visual evoked potentials 3. D. Nerve conduction studies See below for discussion. – occipital). A channel is the voltage difference measured between pairs of these electrodes. A standard EEG recording usually lasts around 30 minutes. An experienced neurophysiology technician can usually obtain a good quality recording from even the most resistant of children. Movement causes artefacts making interpretation difficult. The background EEG recording evolves over the course of childhood. The age of the child is therefore crucial in EEG interpretation. The sleep–wake state and any medications can also affect EEG pattern. EEG wave patterns are classified by frequency into alpha (8–13 Hz), beta (>13 Hz), theta (4–7 Hz) and delta (1–3 Hz). After about the age of 3 years, the predominant EEG rhythm is alpha rhythm. This is most clearly expressed over the posterior regions. Alpha becomes prominent when eyes are closed. Activation procedures Several techniques are used to maximize the amount of information gained during an EEG recording. 1. Hyperventilation – usually for 3 minutes. This causes significant slowing of background activity and can reveal abnormalities not obvious in the resting record. Classically, this technique provokes an absence seizure in children with childhood absence epilepsy. Hyperventilation is achieved by asking the child to blow on a toy windmill. 2. Photic stimulation – involves light flashes at specific frequencies (1–30 Hz) for 5–10 seconds. This can reveal a tendency to photosensitivity, and can allow more careful classification of an epilepsy syndrome. 3. Sleep deprivation – can induce seizure activity in some epilepsy syndromes, e.g. juvenile myoclonic epilepsy. Interictal epileptiform discharges can be activated during sleep, especially in benign localization-related epilepsy syndrome (e.g. BECCTS). If epilepsy is suspected and a standard awake EEG does not provide adequate information to allow classification of the epilepsy syndrome, a sleep EEG may help. Sleep may be achieved by depriving a child of sleep the night before a recording or induced by medication, e.g. melatonin. Peripheral neurophysiology Most peripheral neuropathies in children present with sensory and motor deficits in a symmetrical distal
28 562Neurology Headache Headache is a common symptom, and arises through mechanisms that are not fully understood. Prevalence of headache in children and adolescents is around 50–60%, and that of migraine 7–10%. Below the age of 7 years, headaches are slightly more common in boys than girls, but this ratio reverses with age. In addition, prevalence of headaches also increases with age, reaching adult values in the late teens. A detailed history is the key to identifying the cause of headaches. As well as asking about the site, severity and duration of the headache, one must determine how often they occur, whether the onset is sudden or gradual and if there is any aura. Associated features such as nausea, visual symptoms and weakness might suggest migraine. Precipitants, relieving factors and family history are vital. Figure 28.6 is a simple clinical guide to approaching a child with a headache. The International Headache Society divides headaches into three main groups (Table 28.4). Although severe acute headache can be a symptom of meningeal irritation or raised intracranial pressure, it is more commonly associated with a viral ‘flu-like’ illness. Recurrent and chronic headache can also stem from raised intracranial pressure, but is more commonly due to tension headache or migraine. Rarely, it is a symptom of hypertension. Tension headaches are the most common type of primary headache. These may be episodic or chronic. Chronic daily headache is a less common problem and may in itself be caused by overuse of analgesics, which are helpful in episodic headache but may provoke chronic headache. Migraine Migraine occurs in 10% of children aged 5–15 years. They are slightly more common in boys in early childhood, but by adolescence, girls predominate with a ratio of 3:1. The diagnosis of migraine is based on clinical grounds and summarized in Table 28.5. Acute migraine headaches usually occur when either external or internal triggers are activated to reach a particular threshold. External triggers may include certain foods, internal triggers may include stress or relief from stress. Particular phases of migraine are often described: – Prodrome – prior to onset, vague change in mood. – Αura – usually involves some focal neurological symptoms, often visual disturbance, lasting less than one hour. – Headache phase – often unilateral, can be associated with a feeling of nausea or pattern. Onset is usually gradual. These children should attend a specialized paediatric neuromuscular clinic. Investigation techniques used are detailed below. Electromyography Electromyography (EMG) involves inserting a needle electrode directly into the muscle under investigation. The muscle action potentials are captured on recording equipment and amplified as a sound. Usually there is no electrical activity heard in a resting muscle. Voluntary effort allows action potentials to be recorded. Nerve injury and myopathic conditions will cause fibrillation potentials and characteristic changes in EMG pattern. Myotonia can also be detected and has a characteristic sound. Nerve conduction studies This investigation can confirm a clinical suspicion of a peripheral neuropathy and can allow classification into axonal degeneration or demyelinating conditions. In addition, this study can indicate if both motor and sensory nerve fibres are involved. An electrode records the compound muscle action potential (CMAP) over the desired muscle group. A stimulating electrode is placed at two measured points along the nerve pathway. The latency, amplitude, wave form and velocity of conduction are all recorded. Normal age-specific values allow for comparison. In axonal degeneration, the conduction velocity is usually preserved, but the amplitude of the compound action potential will be reduced. Demyelination usually results in reduced conduction velocities. Visual evoked potentials Visual evoked potentials (VEPs) are simple, noninvasive tests (see Chapter 30, Ophthalmology, for details). They are sometimes appropriate for younger children who cannot easily communicate their visual symptoms or cooperate with the more standard vision assessment techniques. They can help detect lesions of the sensory visual pathways, e.g. demyelination of the optic nerve in optic neuritis. They also allow some degree of quantification and can be a useful technique to monitor progression of visual abnormality. Electroretinogram Electroretinogram evaluates retinal function by placement of an electrode on or very close to the eye in addition to a reference electrode on the forehead (see Chapter 30, Ophthalmology, for details). This study captures both rod and cone responses, depending on whether the retina is light or dark adapted. Light adapted recordings favour cone responses, while dark adaptation allows rod function analysis.
563 CHAPTER TWENTY-EIGHT Fig. 28.6 Clinical decision pathway for headache. (From Levene M. MRCPCH Mastercourse, 2007, Elsevier Churchill Livingstone, with permission.) Headache Afebrile child Blocked shunt Dental caries Migraine Hypertension Focal signs Tumour Abscess Raised ICP Blocked shunt Tumour Intracranial bleed Raised ICP Tumour BIH Hydrocephalus Blocked shunt Focal signs Tumour/SOL Meningism Meningitis Intracranial bleed Normal CNS Normal CNS Abnormal CNS Abnormal CNS Acute and severe Recurrent/ chronic Migraine Aura Hemicranial or frontal Throbbing Nausea or vomiting Lie down in dark room Family history Tension headache No aura Generalized Dull ache, tightness No nausea Continue with activity No family history Febrile child Viral illness Sinusitis Meningitis Table 28.4 International classification of headache disorders Type Primary headache Secondary headache Cranial neuralgias, facial pain and other headaches Aetiology Idiopathic Secondary to another pathology Nerve-related Examples Migraine Tension headache Cluster headache Drug-induced Infective Raised intracranial pressure Brain tumour Vascular event Hypertensive Trigeminal neuralgia Optic neuritis (Source: International Headache Society, 3rd edition.) vomiting, needing to sleep and photophobia. Can last up to 72 hours, depending on treatment strategies. – Resolution phase – non-specific feeling of being unwell. Pathophysiology of migraine The pathophysiology of migraine is complex, and several theories have been proposed over the years, none of which provides a complete answer. There is a clear genetic component, which has been borne out by twin studies and family histories, and the defect has been isolated for one variant of migraine: familial hemiplegic migraine (FHM). FHM is a rare subtype of migraine with aura that has an autosomal dominance inheritance pattern. The defect has been isolated to chromosome 19, with missense mutations for a subunit of a voltage-gated calcium channel. A linkage to this chromosome also appears to occur in some families with more common migraine types. The trigeminovascular system The trigeminovascular system is capable of activating nociceptive neurons on dural blood vessels. This, in turn, releases active substances that affect the vessel wall (including substance P, neurokinin A, calcitonin gene-related peptide, and nitric oxide). These produce dilatation, protein extravasation, and sterile inflammation, thus producing pain. This is unlikely to be the only mechanism of headache, as giving drugs targeting this process does not seem effective in migraine.
28 564Neurology smooth muscle cells in meningeal vessels; however, both 5-HT1D and 5-HT1B receptors can be found in both tissues to some extent, and even in coronary vessels. The trigeminal sensory neurons and meningeal blood vessels are both important in headache generation. Familiar drugs used to treat migraine (the triptans) are selective 5-HT1B/D agonists. They are hypothesized to work by blocking neurotransmission in the trigeminocervical complex and reducing neuropeptide release. Dopamine Dopamine pathway stimulation produces symptoms familiar to migraine sufferers: nausea and vomiting, irritability, low blood pressure, and hyperactive behaviour. Dopamine antagonists do have clinical efficacy in relieving many, but not all, of the symptoms of migraines. Endothelial dysfunction Nitric oxide is a vasodilator and pro-inflammatory agent, in combination with cyclic-GMP, and is implicated in vascular smooth muscle cell dysfunction. Its levels may be elevated in children with migraine. Magnesium Magnesium shows promise as an anti-migraine therapy. Authors have put forward that relative magnesium deficiency triggers a cascade involving platelet aggregation, glutamate release, and vasoconstrictive 5-hydroxytryptamine. Treatment of migraine Relaxation, lifestyle modification, and simple analgesia are recommended as first-line therapy, but when this is not successful other agents are indicated (Table 28.6). Headache and food diaries are used to try and identify any potential triggers, such as: • Sleep deprivation • Stress • Oral contraceptive pill • Menstrual cycle Table 28.5 International headache society criteria for diagnosis of migraine Type Grade Criteria Migraine without aura A At least five attacks fulfilling B to D below B Headache lasting 4–72 hours (2–48 hours in children) C Headache characterized by at least two of the following: 1. Unilateral location 2. Pulsating quality 3. Moderate or severe intensity (inhibits or prohibits daily activity) 4. Aggravated by climbing stairs or similar routine physical activity D Headache accompanied by at least one of the following: 1. Nausea or vomiting, or both 2. Photophobia and phonophobia Migraine with aura A At least two attacks fulfilling B below B Presence of at least three of the following: 1. One or more fully reversible aura symptoms indicating focal cerebral cortical dysfunction or brainstem dysfunction or both 2. At least one aura symptom develops gradually over more than 4 mins, or two or more symptoms occur in succession 3. No aura symptom lasts more than 60 mins. When more than one aura symptom is present, accepted duration is proportionally increased 4. Headache follows aura with a symptom-free interval of less than 60 mins (it may also begin before or simultaneously with aura) Cortical spreading depression Many pathophysiological processes have been implicated in migraine, both vascular and neuronal, and more than one factor may be responsible. The theory of cortical spreading depression represents a wave of neuronal excitation in the cortical grey matter that spreads from its site of origin. This phenomenon is felt to account for the pre-migrainous aura reported by up to 30% of patients. Serotonin 5-hydroxytryptamine (5-HT) receptor is widely believed to be a central player in migraine. The receptor subtype 5-hydroxytryptamine-1D (5-HT1D) is found in higher numbers in trigeminal sensory neurons, including peripheral projections to the dura and within the trigeminal nucleus caudalis and solitary tract. 5-HT1B receptors are located primarily on Table 28.6 Anti-migraine therapy Non-pharmacological Acute attack Prophylaxis Relaxation Lifestyle modification Avoidance of triggers Biofeedback Occipital nerve stimulation Paracetamol NSAIDs (Opiods) Antiemetics Triptans (Ergotamine) (Dopamine antagonists) Pizotifen Propranolol (Topiramate) (Calcium channel blockers)
565 CHAPTER TWENTY-EIGHT • Vasodilator therapy • Food (e.g. cheese, chocolate) Pharmacological therapy for migraine Acute attacks Analgesics: Paracetamol and non-steroidal antiinflammatories (NSAIDs) are first-line therapy, with opiods reserved for resistant cases. Antiemetics: Domperidone, phenothiazines, and antihistamines may be given separately or in combination with other drugs. They act on receptors including 5-HT3, muscarinic acetylcholine, histamine 1, and dopamine 2 receptors. These receptors are all involved in the areas of the body known to trigger nausea and vomiting: the vestibular nuclei, nucleus of the solitary tract, chemoreceptor vomiting trigger centre, the vomiting centre (lower medulla, reticular formation, dorsal vagal nucleus), and higher brain centres. They act to relieve the nausea associated with migraine attacks. Ergotamine (not recommended in children): Ergotamine acts via serotonin receptor activation, particularly 5-HT1B and 1D receptors, and to a lesser extent at 5-HT1F receptors. Ergot also is a potent alphaadrenergic receptor agonist, which triggers arterial and venoconstriction. This vasoconstriction can affect systemic blood vessels, including the coronary arteries. It carries risk of other side effects due to action at 5-HT1A receptors (nausea, mood change), 5-HT2A receptors (peripheral vasoconstriction), and dopaminergic D2 receptors (nausea and vomiting). Triptans (not licensed below 6 years): Triptans also are agonists at 5-HT1B and 1D receptors, and to a lesser extent at 5-HT1F receptors. 5-HT1B receptors are preferentially expressed in intracranial extracerebral arteries, whilst in the peripheral arteries there are more 5-HT2A receptors. As the triptans are agonists at 5-HT1B receptors, but not at 5-HT2A receptors, they have less systemic vasoconstrictive impact than ergotamine. They are particularly useful in older children with infrequent migraine, as they act quickly and can be taken orally, by subcutaneous injection or intranasally (unlicensed). A non-oral route is useful if nausea and vomiting are prominent features. If a child does not respond to one agent, a trial of a different triptan is advised. They are, however, not indicated for hemiplegic, basilar, or ophthalmoplegic migraine. Regularly taking agents for acute attacks has limitations, as they can produce rebound headache and take time to abort the headache. In those children where significant time off school is occurring or when headaches are very severe, a trial of prophylaxis is warranted. This approach is also indicated if the acute agents are contraindicated or not tolerated, or if the child is considered too young for use of the available licensed agents. Migraine prophylaxis Pizotifen (not licensed below 5 years): Pizotifen is a cycloheptathiophene derivative structurally related to cyproheptadine and the tricyclic antidepressants. It is a serotonin antagonist at 5-HT2A and 5-HT2C receptors, as well as a histamine antagonist and has weak anticholinergic activity. It may act by inhibiting the reuptake of serotonin by blood platelets, preventing loss of tone of extracranial vessels. Propranolol (licensed for use in 2 years and over): A non-selective beta-1 receptor antagonist, propranolol can reduce both the frequency and severity of attacks. Similar beta blockers that also have intrinsic sympathomimetic properties (e.g. pindolol) are ineffective in migraine prophylaxis. Possible mechanisms or therapeutic action include: • Alteration of 5-HT synthesis • Antagonism of 5-HT receptors • Decreased neuronal firing of noradrenergic neurons in the locus coeruleus • Alteration of GABA-mediated firing of periaqueductal grey matter neurons Propranolol should not be given to patients with asthma. Topiramate (not currently licensed in children): An anticonvulsant, topiramate is a sodium channel blocker (as are many antiepileptics), promotes GABAA receptor activation, inhibits L-type high-voltage calcium ion channels, and inhibits glutamate receptors. It is not known which of these mechanisms is responsible for its benefit in migraine. Calcium channel blockers (not currently licensed in children): Drugs such as verapamil or nifedipine act to block calcium channels on the cell wall of arterial blood vessels in the brain. They are also thought to have an impact on the serotoninergic system of the brain. Secondary headaches Headache secondary to underlying disease is uncommon, but must not be overlooked. Particular features on history and examination that may suggest pathology are outlined in Table 28.7. Any child presenting with headache should be assessed with these symptoms or signs in mind, particularly those suggesting raised intracranial pressure. Idiopathic intracranial hypertension Idiopathic intracranial hypertension (IIH) is a syndrome characterized by increased intracranial pressure (ICP) of unknown cause. It was previously referred to as benign intracranial hypertension, but this term is no longer used as, although it is usually self-limiting,
28 566Neurology Cranial neuralgias, facial pain and other headaches Trigeminal neuralgia is characterized by sudden, sharp ‘electric shock’ pain that is severe and affects one side of the face in the distribution of the trigeminal nerve. It can occur several times per day, and be triggered by movement or by light touch of the face, even a gust of wind. Commoner in women and those over 50 years of age, it can be treated using carbamazepine. Pathogenesis is thought to be related to vascular compression at the trigeminal nerve entry into the pons, resulting in focal trigeminal nerve demyelination. Surgery to release the pressure can be effective but requires detailed consideration. Epilepsy Neuronal transmission occurs via the neuronal action potential and dysfunction in this most basic of physiological mechanisms is the fundamental basis of epilepsy and seizure disorders. Action potentials occur due to depolarization of the neuron cell membrane, with the wave of depolarization propagating along the axon to cause release of neurotransmitters at the axon terminal. Action potentials occur in an all-or-none fashion as a result of local changes in membrane potential caused by a net positive influx of ions. Membrane potential thus varies with activation of various ligand-gated channels. These are affected by binding to neurotransmitters or by changes in transmembrane potentials. A cellular hyperexcitable state can result from increased excitatory factors, decreased inhibition, an alteration in voltage-gated ion channels, or a change in ion concentrations, all of which favour membrane depolarization. Neurotransmitters are released by the presynaptic terminal at the synapse. They then bind to specific receptors on the postsynaptic membrane for that ligand. Ligand binding causes channel activation and movement of ions in or out of the cells. The major neurotransmitters in the brain include glutamate, gamma-aminobutyric acid (GABA), acetylcholine (ACh), noradrenaline (norepinephrine), dopamine and serotonin. Other molecules including neuropeptides and hormones are also thought to modify neurotransmission but over longer time periods. The major excitatory neurotransmitter is glutamate. There are several subtypes of glutamate receptors (e.g. AMPA and NMDA) which are permeable to sodium and potassium. It is the movement of Na+ and K+ through the channels which leads to depolarization and generation of an action potential. The complex mechanism of channel function regulating neuron action potentials is the fundamental it can last several months and it may lead to permanent visual loss. The pathophysiology of this condition remains unclear. By definition, the elevated ICP is not related to an intracranial disorder, a meningeal process, or cerebral venous thrombosis. Many theories have been suggested, including: increased brain water content, excess CSF production, reduced CSF absorption, and increased cerebral venous pressure. A fundamental problem is our lack of understanding of the mechanism for control of normal CSF volumes and pressures. The condition is particularly encountered in overweight or obese teenage girls and young women. The role of excess fat tissue in modifying endocrine function is under investigation and may contribute at least in part to this disorder. It causes headache, nausea, and visual disturbance, with blurred vision, visual loss or diplopia, which can be irreversible. Papilloedema is present on fundoscopy. Brain imaging is normal, but the opening CSF pressure on lumbar puncture is raised. Management may involve therapeutic CSF removal by repeat lumbar punctures, taking off enough fluid each time to reduce intracranial pressure. Other options include use of the drug acetazolamide, as this is a carbonic anhydrase inhibitor that alters flow of hydrogen, sodium, potassium, bicarbonate, and water. It is also used as a diuretic, glaucoma treatment, and metabolic acidifying agent; it works in IIH to reduce CSF production. Cases non-responsive to the above measures may progress to surgery for CSF shunt insertion or optic nerve sheath fenestration. Table 28.7 Features suggestive of sinister underlying pathology in a child with headache History Examination Recent onset Increasing in severity Worse in the morning, on lying down, or on coughing* Associated with vomiting, often without nausea in the early stages* Seizures Deterioration in school performance* Change in personality or behaviour* Lethargy/drowsiness ‘Irritability’ Papilloedema* Bitemporal hemianopia (craniopharyngioma) or other visual problems Endocrine dysfunction (diabetes insipidus, hypothyroid, growth failure, early or late puberty) Abnormal neurological examination Ptosis, gaze palsy Abnormal head position Raised blood pressure, slow pulse and widened pulse pressure* A ‘tense’ or ‘full’ fontanelle (infants)* Rapidly increasing head circumference (infants)* *Particularly associated with raised intracranial pressure of any cause.
567 CHAPTER TWENTY-EIGHT calcium channel subunit) are associated with migraine disorders such as hemiplegic migraine. Pharmacological treatment of epilepsy Choice of anticonvulsant therapy is dependent upon seizure type and epilepsy syndrome (Table 28.8). Monotherapy at the lowest dose to achieve seizure frequency is the aim of treatment. Therapy is usually continued for a minimum period of two years after achieving seizure freedom. The availability of a palatable liquid, granules, or dissolvable preparation is of prime importance among the paediatric population. Some antiepileptic medications such as sodium valproate, phenytoin and carbamazepine have been in use for a great many years. More recently, some basis of epilepsy (and other neurological conditions). Situations leading to altered brain tissue physiology, such as infection, inflammation or vascular compromise, will lead to channel dysfunction, thus seizure activity often results with various other neurological symptoms. Similarly, there are an increasing number of recognized genetic channelopathies causing malfunction of particular ion channels. Such conditions often have epilepsy as part of the phenotypical presentation. A good example is the SCN1A sodium channelopathy. Mutations in this gene often present with troublesome seizures. The range of seizure presentations is variable, from frequent febrile seizures, the syndrome of generalized epilepsy with febrile seizures plus (GEFS+), as well as severe myoclonic epilepsy of infancy. In addition, mutations in some genes (e.g., CACNA1A – a Table 28.8 Recommended anticonvulsants by seizure type Seizure type First line Adjunct May worsen seizures Generalized tonic–clonic Lamotrigine Sodium valproate Levetiracetam Clobazam Lamotrigine Levetiracetam Sodium valproate Topiramate Carbamazepine Gabapentin Oxcarbazepine Phenytoin Pregabalin Tiagabine Vigabatrin Tonic or atonic Sodium valproate Lamotrigine Carbamazepine Gabapentin Oxcarbazepine Pregabalin Tiagabine Vigabatrin Absence Ethosuximide Lamotrigine Sodium valproate Carbamazepine Gabapentin Oxcarbazepine Phenytoin Pregabalin Tiagabine Vigabatrin Myoclonic Levetiracetam Sodium valproate Topiramate Clobazam Clonazepam Piracetam Carbamazepine Lamotrigine Oxcarbazepine Phenytoin Pregabalin Tiagabine Vigabatrin Focal Carbamazepine Lamotrigine Levetiracetam Oxcarbazepine Sodium valproate Carbamazepine Lamotrigine Levetiracetam Oxcarbazepine Lacosamide Sodium valproate Topiramate Status epilepticus Buccal midazolam Rectal/intravenous diazepam Intravenous lorazepam Intravenous phenobarbital Phenytoin (Adapted from NICE guidance.)
28 568Neurology Lamotrigine – also acts at voltage-dependent sodium channels, but is believed to have other modes of actions. The major side effect of this drug is development of a rash, which can progress to a Stevens–Johnson syndrome. The risk can be reduced by very gradual titration of dose. This slow introduction does, however, limit its usefulness in the acute setting. Plasma levels are significantly elevated by sodium valproate, so doses must be reduced appropriately. Oxcarbazepine – a compound with a similar chemical structure to carbamazepine and likely a similar mechanism of action. Zonisamide – derived from the sulfonamide group of drugs and is unrelated to other anticonvulsants. The main mechanism of action seems to be at voltage-dependent sodium channels as well as calcium channels. Side effects include behavioural change, drowsiness and dizziness. Renal stones have also been reported. Lacosamide – a newer drug that enhances slow inactivation of voltage-dependent sodium channels resulting in the inhibition of repetitive neuronal firing. This medication is usually well tolerated but experience is more limited at present. Rufinamide – structurally unrelated to other epilepsy drugs and modulates the activity of sodium channels, prolonging their inactivation phase. The most common side effects are sleepiness and vomiting. Medications acting on calcium currents Ethosuximide – reduces calcium channel currents in thalamic neurons, which are thought to have a role to play in absence type seizures. The major side effects include nausea, vomiting, sleep disturbance, drowsiness, and hyperactivity. Because of its distinctive taste, compliance can be an issue. Medications affecting GABA systems Gamma-aminobutyric acid (GABA) is a neurotransmitter that is widely distributed throughout the central nervous system and exerts postsynaptic inhibition. Phenobarbitone – among the oldest AEDs still in current use. It is effective for the management of both generalized and partial seizures. The main side effect, which limits its use, is sedation. Tiagabine – a second generation AED that is indicated as adjunctive treatment for partial seizures. Its use in the paediatric setting is limited to date. newer drugs have become available. The mechanism of action of most medications is reasonably well understood. Knowing how a particular antiepileptic drug (AED) works can be helpful in determining an alternative treatment if an effective medication is not tolerated, or deciding upon a therapeutic strategy if a particular class of AED has not been clinically effective. Question 28.10 Mechanism of action of AEDs Concerning the mechanism of action of antiepileptic medications, which of the following agents does NOT work on the voltage-dependent sodium channel? Select ONE answer only. A. Carbamazepine B. Ethosuximide C. Lacosamide D. Lamotrigine E. Phenytoin Medications acting at voltagedependent sodium channels Carbamazepine – binds to voltage-dependent sodium channels extending the inactivated phase and thus preventing the generation of rapid action potentials. It is usually well tolerated but may cause systemic upset in the form of nausea, vomiting and diarrhoea. Central side effects include drowsiness, headache and dizziness. A Stevens–Johnson type adverse reaction is possible and more likely in certain ethnic groups. Phenytoin – has been widely used for a great many years. It acts on both voltage-dependent sodium channels and on sodium–potassium ATPase and thereby reduces synaptic transmission. Phenytoin is an integral step in the management of status epilepticus in Advanced Paediatric Life Support (APLS) guidelines. Drug interactions are common. Very few children will be maintained on longterm phenytoin due to the side-effect profile, which includes gum hypertrophy, rash and excess hair growth. Answer 28.10 B. Ethosuximide. See below for discussion.
569 CHAPTER TWENTY-EIGHT also weakly inhibits carbonic anhydrase in the central nervous system. Weight loss is a common side effect. Levetiracetam – a broad-spectrum drug. The mechanism of action for this medication is unknown. It is generally well tolerated. Oral and intravenous administration is possible, with relatively fast escalation. Side effects include behavioural change and sleepiness. Non-pharmacological treatment of epilepsy Vagus nerve stimulation Vagus nerve stimulation (VNS) is an alternative management strategy for patients with refractory epilepsy. Initial animal studies indicated that stimulation of the vagus nerve could be used to terminate seizures. Although the exact mechanism of vagus nerve stimulation is not well understood, it probably relates to the complex connections between the vagus nerve and various regions of the brainstem, midbrain and cortex. The device is implanted on the chest wall with electrodes attached to the left vagus nerve in the neck. Onset of benefit can take many months to emerge. MRI scanning is not possible with VNS in situ. Ketogenic diet The ketogenic diet is a high-fat, low-carbohydrate, adequate protein diet that has been used in the treatment of difficult epilepsy for many years. This diet regime causes production of ketone bodies (β-hydroxybutyrate, acetone and acetoacetate) – from fatty acid oxidation by the liver – and reduced blood glucose levels. The ketogenic diet is also the treatment of choice for a small number of other neurometabolic conditions, including GLUT1 deficiency, and PDH deficiency. Elevated free fatty acids lead to chronic ketosis and increased concentrations of polyunsaturated fatty acids in the brain. Chronic ketosis is predicted to lead to increased levels of acetone; this may activate potassium channels to hyperpolarize neurons and limit neuronal excitability. Chronic ketosis is also felt to alter brain biochemistry to promote inhibitory neurotransmitter levels. This actual scientific basis is not fully understood but is likely to involve many pathways, including free radical generation, interleukin and cytokine balance, and various mitochondrial pathways, ultimately leading to reduced membrane hyperexcitability, and thus improved seizure control. Careful dietetic planning and monitoring is required, and all children need an individualized care plan in case of illness or hospital admission. Vigabatrin – an irreversible inhibitor of GABAtransaminase, which increases the concentration of GABA in the central nervous system. It is particularly effective for infantile spasms in children with tuberous sclerosis. The major and significant side effect is an irreversible visual field loss on prolonged use. Therefore, it is rarely used for more than 6 months. Assessing visual fields is very difficult in younger children. Benzodiazepines – enhance GABA inhibition by increasing the frequency of GABA-mediated chloride channel openings. This group of medicines is often associated with the development of tolerance, which significantly limits their use in the longer term. Sudden discontinuation may lead to withdrawal seizures and significant behavioural change in children. Diazepam and lorazepam – often used in the acute setting. Clobazam, clonazepam and nitrazepam are sometimes used in the longer term, or for repeated short courses, either when seizures are troublesome or when background medications are being altered. Common side effects include sedation, as well as drooling, insomnia, and behavioural change. Medications acting at glutamate receptors Glutamate is the most prevalent excitatory neurotransmitter. Perampanel – a relative newcomer, which targets post-synaptic AMPA receptors, and is only licensed in children over 12 years. Experience is limited. Mood change, fatigue and headache have been reported as side effects. Medications with other mechanisms of action A number of AEDs have multiple mechanisms by which they prevent seizures. Sodium valproate – a broad-spectrum AED used alone and in combination for the treatment of generalized and partial seizures. It has been in mainstream use for many years. It is known to act at voltage-dependent sodium channels, as well as increasing gamma-aminobutyric acid (GABA). It is also thought to act against certain calcium channels. Side effects include nausea, vomiting, hair loss and weight gain. Use of this medication is teratogenic. This must therefore be taken into account in teenage girls. Topiramate – blocks voltage-dependent sodium channels, promotes activity of GABA receptors, and antagonizes an NMDA–glutamate receptor. It
28 570Neurology Epilepsy Society. <http://www.epilepsysociety.org.uk>; [accessed 01.09.15]. The Epilepsy Society has a range of useful information available on its website. International Headache Society. <http://www.ihsheadache.org>; [accessed 01.09.15]. Detailed resource on all aspects of headache diagnosis and management. International League Against Epilepsy. <http://www.ilae.org>; [accessed 01.09.15]. Excellent comprehensive website on all aspects of epilepsy. Particularly useful classification section, which is regularly updated. International Parkinson and Movement Disorder Society. <http://www.movementdisorders.org>; [accessed 01.09.15]. All movement disorders are addressed in this site, with links to additional resources. National Institute for Health and Care Excellence. The epilepsies: the diagnosis and management of the epilepsies in adults and children in primary and secondary care. NICE guidelines [CG137]; 2012. Neuromuscular Disease Center (Washington University). <http://neuromuscular.wustl.edu>; [accessed 01.09.15]. All aspects of neuromuscular disease are covered in this website, with particular emphasis on genetics. The development of the nervous system. <https:// www.youtube.com/watch?v=Cu4lQYbOzzY>; [accessed 01.09.15]. Animated presentation showing the embryology of the central nervous system, derived from Moore KL, Persaud TVN, Torchia MG. Before we are born: essentials of embryology and birth defects, 8th ed. Philadelphia: Saunders, Elsevier; 2012. Wöber-Bingöl C. Epidemiology of migraine and headache in children and adolescents. Curr Pain Headache Rep 2013;17(6):341. Further reading Abu-Arafeh I, Razak S, Sivaraman B, Graham C. Prevalence of headache and migraine in children and adolescents: a systematic review of population-based studies. Dev Med Child Neurol 2010;52(12):1088–97. Blume H. Pediatric headache: a review. Pediatr Rev 2012;33(12):562–76. Child Neuro Channel. <http://childneurochannel.org/ disorders>; [accessed 01.09.15]. Range of child neurology topics written by professionals. Recent scientific advances which have improved clinical practice – epilepsy Whilst drug treatments for epilepsy tend to filter down through adult neurology prior to being introduced in children, the introduction of the ketogenic diet and vagal nerve stimulation (VNS) has permitted previously intractable cases to achieve a measure of control. Stereotactic surgery is now potentially curative for certain focal epilepsies, and wider experience and availability of specialist scanning techniques such as positron emission technology (PET) offers hope of understanding much more about the human brain and its disorders.
LEARNING OBJECTIVES By the end of this chapter the reader should: • Understand the biochemistry of metabolism, including urea cycle, Krebs cycle and fatty acid cycle • Understand the pathophysiology of metabolic disorders, e.g. electrolyte and acid– base disturbance, hyperammonaemia and hypoglycaemia • Know the genetic and environmental factors in the aetiology of metabolic disorders • Be aware of the metabolic disorders identified on neonatal screening • Understand the investigations that are used to diagnose metabolic disorders • Understand the principles of dietary and pharmacological treatment of metabolic diseases 571 CHAPTER TWENTY-NINE Introduction Intermediary metabolism is the term given to the biochemical reactions that degrade, synthesize or interconvert molecules within the cells. There are numerous metabolic pathways, which serve the following aims: • Generation of energy • Catabolism of organic molecules • Synthesis of cellular building blocks • Excretion of harmful substances. These pathways require enzymes, which, if absent or deficient, can give rise to an inborn error of metabolism (IEM). This chapter describes the key metabolic pathways and links them with their associated diseases. Acid–base disturbance Definitions The key terms are outlined in Table 29.1. Biochemistry Acid–base balance is essential for correct cellular functioning. Blood gas measurement can identify the primary disturbance (Table 29.2). In general: • Metabolic disturbances are compensated acutely by changes in ventilation and chronically by renal responses • Respiratory disturbances are compensated by renal responses. In the case of metabolic acidosis, calculation of the anion gap will determine if there is the presence of an unmeasured anion such as an organic acid, e.g. methylmalonic or propionic acid (Table 29.3 and Fig. 29.1). Acidosis with a normal anion gap is often associated with hyperchloraemia because the loss of base is buffered by an increase and/or retention of chloride. Clinical Metabolic acidosis is a common finding. In the majority of cases, it reflects severe illness rather than an inborn error of metabolism (IEM). The latter should be considered if the acidosis is out of keeping with the clinical picture, is persistent despite standard management and there is no identifiable acid present, e.g. lactate or ketones. Presentation Metabolic acidosis is typically non-specific in presentation. Signs may include a reduced conscious level, Elisabeth Jameson Metabolic medicine C H A P T E R 29
29 572Metabolic medicine vomiting or those associated with the underlying aetiology, e.g. non-blanching rash in the case of meningococcal sepsis. Many patients will display an increased respiratory rate, Kussmaul respiration, reflecting the compensatory hyperventilation that occurs to promote removal of carbon dioxide. Diagnosis The blood gas is key to identifying the primary disturbance in acid–base balance. In addition to calculating the anion gap, ketones and lactate should be measured as potential causes of acidosis. When investigating for an IEM, urine organic acids and plasma amino acids and acylcarnitines are required. It is important to measure an ammonia level as this can be elevated in an organic acidaemia due to the metabolites inhibiting the urea cycle. Management The underlying aetiology, when known, should be treated. If acidosis is severe, normalization of acid– base balance can be achieved with administration of sodium bicarbonate. Lactic acidosis Normal plasma lactate is <2 mmol/L. A raised level has a wide differential (Table 29.4). In terms of IEM, mitochondrial disorders are classically associated with a raised lactate, with levels often fluctuating. When considering the possibility of mitochondrial disease, measuring cerebral spinal fluid for a raised level can be helpful. However, a normal lactate does not exclude a mitochondrial disorder. Table 29.1 Acid–base definitions Terminology Definition Acid A proton or hydrogen ion donor. It can dissociate to yield H+ and the corresponding base. Anion gap (see Fig. 29.1) [Na+ + K+ ] − [Cl− + HCO3 − ]. Normal = 10 − 16 mmol/L. Reflects concentration of those anions not routinely measured, e.g. organic acids (see Question 29.1). Base A proton or hydrogen ion acceptor. Can accept H+ to form corresponding undissociated acid. Base excess Measures the change in the concentration of a buffer base from the normal value. Normal range = +/− 2 mmol/L. Buffer Consists of a weak acid in the presence of its base. A buffer serves to minimize changes in H+ concentration in response to the addition of an acid or base. Examples of buffers in: Plasma – bicarbonate, proteins, inorganic phosphate (Pi) Erythrocytes – haemoglobin, bicarbonate, Pi Kidneys – bicarbonate, Pi, ammonium pH The logarithm to the base 10 of the reciprocal of the hydrogen ion concentration. pH = −log [H+ ] pKa The pH of a buffer at which half the acid molecules are undissociated and half are associated. Table 29.2 Acid–base disturbance Abnormality Primary disturbance Effect on Base excess Compensatory response pH pCO2 Respiratory acidosis ↑ pCO2 ↓ ↑ Negative ↑ [HCO3 − ] Metabolic acidosis ↓ [HCO3 − ] ↓ N or ↓ Negative ↓ pCO2 Respiratory alkalosis ↓ pCO2 ↑ N or ↓ Positive ↓ [HCO3 − ] Metabolic alkalosis ↑ [HCO3 − ] ↑ N or ↑ Positive ↑ pCO2 Table 29.3 Metabolic acidosis and anion gap With normal anion gap With raised anion gap • Intestinal loss of base, e.g. diarrhoea, fistulae • Renal loss of base, e.g. renal tubular acidosis (RTA) types 1 and 2, pyelonephritis • Carbonic anhydrase inhibitors • Diabetic ketoacidosis • Renal failure • Poisoning with: salicylate, methanol, propylene glycol, iron, isoniazid, ethylene glycol • Inborn errors of metabolism, e.g. organic acidaemia, lactic acidosis Table 29.4 Causes of a raised lactate Metabolic Non-metabolic Respiratory chain disorder Pyruvate dehydrogenase deficiency Pyruvate carboxylase deficiency Disorders of gluconeogenesis Glycogen storage disorders Organic acidaemia Fatty acid oxidation disorder Hypoxic–ischaemic encephalopathy Severe illness Cardiac disease Sampling artefact
573 CHAPTER TWENTY-NINE Fig. 29.1 Representation of the anion gap, an estimate of the osmolar difference between measured cations and anions. Na+ Anion gap Organic acids HCO3– CI– Inorganic acids Proteins Measured cations Measured anions Unmeasured cations Unmeasured anions K+ Ca++ Mg++ 150 mmols/L Question 29.1 A 6-day-old baby with tachypnoea A 6-day-old 3 kg term baby boy, born after a normal pregnancy and delivery, presents with reduced feeding and tachypnoea (respiratory rate 80/minute) over the last 24 hours. On examination, he is encephalopathic. Investigations: Blood: Full blood count mild pancytopenia Sodium 136 mmol/L Potassium 3.6 mmol/L Chloride 110 mmol/L Lactate 8 mmol/L (1–2.8) Ammonia 60µmol/L (normal <100) C-reactive protein 6 mg/L Blood gas: pH 7.29 pCO2 2.0kPa (15 mmHg) pO2 13kPa (98 mmHg) Bicarbonate 10 mmol/L Base excess −18 mmol/L Which of the following is the most likely diagnosis? Select ONE answer only. A. Group B streptococcal septicaemia B. Hypoxic–ischaemic encephalopathy C. Organic acid disorder D. Surfactant protein B deficiency E. Urea cycle defect Key points – organic acid disorders • Methylmalonic and propionic acidaemia are the most common organic acidaemias • Can cause pancytopenia because of effects on the bone marrow at times of decompensation • pH can be maintained with hyperventilation • A lactate of 8 mmol/L would not by itself generate such a large anion gap • To calculate a half correction (using this case as an example): Base deficit weight kg mmol NaHCO × × = × × = − ( ) . . . 0 3 2 18 3 0 3 2 8 1 3 Answer 29.1 C. Organic acid disorder. There is a marked anion gap. The anion gap = (136 + 3.6) − (110 + 10) = 31.6 mmol/L. In this patient, the gas normalizes with intravenous 10% dextrose and two half corrections of sodium bicarbonate. Further investigations: urine organic acid analysis reveals methylmalonic acidaemia (MMA). Group B streptococcal septicaemia is possible, but is more likely to present with shock and a much more abnormal blood count, including low or high white blood cell count and thrombocytopenia. The low CRP in spite of being ill for 24 hours is also against this diagnosis. Hypoxic–ischaemic encephalopathy would present before 6 days. Surfactant protein B deficiency would present with increasing respiratory distress from birth. A urea cycle defect is possible, but the ammonia level is normal for a neonate.
29 574Metabolic medicine The urea cycle and hyperammonaemia Biochemistry Ammonia Ammonia is a highly neurotoxic chemical detoxified by the urea cycle (Fig. 29.2), which principally occurs in the liver. Ammonia is formed from: • Nitrogen produced from amino acid metabolism • Glutamate by the action of glutamate dehydrogenase • Glutamine by the action of glutaminase. • Alanine and glutamine produced by muscle turnover • Urease-positive gut bacteria • Ingested protein not utilized in biochemical processes The urea cycle The urea cycle (see Fig. 29.2) consists of six enzymes, with each full cycle disposing of two nitrogen atoms: Fig. 29.2 The urea cycle. 1, N-acetylglutamate synthetase (NAGS); 2, carbamoyl phosphate synthetase 1 (CPS1); 3, ornithine transcarbamoylase (OTC); 4, argininosuccinate synthetase (ASS); 5, argininosuccinate lyase (ASL); 6, arginase; 7, mitochondrial ornithine transporter; 8, mitochondrial aspartate–glutamate carrier (citrin); 9, glutamate dehydrogenase; 10, glutaminase; 11, glutamine synthetase. Glutamine Glutamate Mitochondrial matrix Cytosol Urea Ornithine Arginine Acetyl-CoA Argininosuccinate Orotic acid Citrulline Aspartate Carbamoyl phosphate N-acetyl glutamate NH3 ATP + HCO3 Aspartate Ornithine Citrulline 11 10 1 9 2 3 4 6 7 5 8 one from ammonia and one from aspartate. The cycle progresses as: • N-acetylglutamate forms from the condensation of glutamate with acetyl-CoA catalysed by N-acetylglutamate synthetase • Condensation of ammonia with bicarbonate forms carbamoyl phosphate catalysed by carbamoyl phosphate synthetase. The latter is only active in the presence of N-acetylglutamate. • Carbamoyl phosphate condenses with ornithine to form citrulline • Citrulline is transferred into the cytoplasm and combines with aspartate to form argininosuccinate, catalysed by argininosuccinate synthase. • Argininosuccinate lyase cleaves argininosuccinate to arginine • Arginine is hydrolysed to urea, which is excreted in urine. Each urea molecule contains two nitrogen atoms and one carbon atom. Ornithine is transported back into the mitochondrion by the ornithine transporter.
575 CHAPTER TWENTY-NINE (see Genetics of metabolic disorders, below). Male infants are severely affected and many do not survive the neonatal period. Female carriers have a varied phenotype; the majority remain asymptomatic but approximately 15% will require treatment. Diagnosis Diagnosis of urea cycle disorders (Table 29.6) is based upon plasma amino acid analysis and the presence or absence of urine orotic acid, which is produced when carbamoyl phosphate passes into the pyrimidine pathway. The absence of orotic acid in a urea cycle disorder implies N-acetylglutamate synthetase (NAGS) or carbamoyl phosphate synthetase (CPS) deficiency. Orotic acid is classically very elevated in OTC because of the accumulation of intracellular carbamoyl phosphate. The remaining defects are associated with a much smaller or negligible amount of orotic aciduria. Management This can be thought of in terms of acute and long term. Acute: • Stop feeds and commence 10% dextrose to reduce nitrogen load on the cycle • Commence intravenous ammonia scavenging medications (see Principles of pharmacological treatment, below) • Commence intravenous arginine to replenish the urea cycle • Transfer to specialist centre in preparation for haemofiltration Chronic: • Low protein diet to reduce nitrogen load on the cycle • Ammonia scavenging medications to aid excretion of excess nitrogen • Arginine (except in arginase deficiency) to replace arginine not produced by the urea cycle Ammonia is also buffered by the conversion of glutamate to glutamine via the action of glutamine synthetase. At times of hyperammonaemia, the glutamine concentration increases and thus can be used as an indicator of insufficient urea synthesis and is indicative of longer term metabolic control. Clinical Hyperammonaemia (normal plasma ammonia levels are <100µmol/L in neonates and <50µmol/L thereafter) has a wide differential (Table 29.5). Urgent measurement of ammonia should therefore take place in any baby, child or adult presenting with unexplained encephalopathy or illness. The urea cycle disorders (UCD) arise due to deficiency of one of the six main urea cycle enzymes. Presentation The classic presentation is the term baby who becomes increasingly sleepy and encephalopathic on day 3–5 of life with poor feeding and vomiting (see Question 29.2). Ammonia levels can rise rapidly. Urgent investigation is required to clarify the diagnosis and guide management. The urea cycle disorders are inherited in an autosomal recessive manner, except for ornithine transcarbamylase (OTC) deficiency, which is X-linked Table 29.5 Differential diagnosis of hyperammonaemia Inborn errors of metabolism Acquired Urea cycle disorder Organic acidaemia Fatty acid oxidation disorders Pyruvate carboxylase deficiency Ornithine aminotransferase deficiency HHH syndrome (hyperammonaemia, hyperornithinaemia, homocitrullinuria) Transient hyperammonaemia of newborn Severe illness Herpes simplex infection Cardiac disease Medications (sodium valproate, asparaginase) Reye-like illness Liver disease Porto-systemic shunts Artefactual from poor sampling Table 29.6 Diagnosis of urea cycle disorders Enzyme Disorder Plasma amino acid concentrations relative to reference range Urine orotic acid Alanine Glutamine Citrulline ASA Arginine NAGS NAGS def ↑ ↑ Normal CPS CPS def ↑ ↑ ↓ ↓ Normal OTC OTC def ↑ ↑ ↓ ↓ ↑↑↑ ASS Citrullinaemia ↑ ↑ ↑↑ ↓ ↑ ASL ASA def ↑ ↑ ↑ ↓ ↑ Arginase Arginase def ↑ ↑ ASA, argininosuccinic aciduria; ASL, argininosuccinate lyase; ASS, argininosuccinate synthetase; CPS, carbamoyl phosphate sythetase; def, deficiency; NAGS, N-acetylglutamate synthetase; OTC, ornithine transcarbamylase.
29 576Metabolic medicine Glucose and glycogen metabolism Definitions There are a number of key processes in this pathway, as defined in Table 29.7. Glucose biochemistry (Fig. 29.3) Glucose Glucose provides an immediate energy source via its conversion into pyruvate with the net production of 2 molecules of ATP and NADH (nicotinamide adenine dinucleotide) per glucose molecule. Excess glucose can be converted into glycogen, which can be utilized Table 29.7 Definitions in relation to glucose and glycogen metabolism Term Definition Hypoglycaemia A true blood glucose of <2.6 mmol/L. Gluconeogenesis Synthesis of glucose from non-glucose precursors, i.e. amino acids (except leucine and lysine) and glycerol in the liver, kidney or intestinal epithelium. Glucose can also form from intermediates of glycolysis, the Krebs cycle and fructose. Glycolysis Oxidation of glucose to pyruvate with generation of adenosine triphosphate (ATP). Glycogenesis Conversion of excess glucose to glycogen. Key enzymes are glycogen synthase and the branching enzymes. Glycogenolysis Degradation of glycogen to glucose. Phosphorylase is the controlling enzyme. Key points – urea cycle defects • Ammonia is a respiratory stimulant and can cause respiratory alkalosis • Seizures can be seen in the acute phase due to cerebral oedema secondary to the effects of hyperammonaemia • Early referral to PICU for haemofiltration is essential • High ammonia levels (>1000µmol/L) are associated with poor prognosis in terms of survival and long-term neurological outcome Question 29.2 A 5-day-old baby with drowsiness and poor feeding A 5-day-old baby girl is born at term after a normal pregnancy and delivery. She presents with a 24-hour history of increasing sleepiness and poor feeding. On examination, she is encephalopathic with an irritable cry. Investigations: Blood: Haemoglobin 136 g/L White cell count 10.0 × 109 /L Platelets 360 × 109 /L CRP 10 mg/L Glucose 4.0 mmol/L Ammonia 875µmol/L (<100) Lactate 5 mmol/L (1–2.8) Urea and electrolytes normal Liver function tests normal Calcium, phosphate, ALP normal Blood gas: pH 7.5 pCO2 2.5kPa pO2 11.3kPa Base excess −5 mmol/L Bicarbonate 22 mmol/L Which of the following is the most likely diagnosis? Select ONE answer only. A. Hyperinsulinaemia of the newborn B. Intrapartum hypoxia C. Pyridoxine dependency D. Septicaemia E. Urea cycle defect Answer 29.2 E. Urea cycle defect. A urea cycle defect is the most likely because of the elevated lactate level and the extremely high ammonia. Hyperinsulinaemia is possible but ruled out by the normal blood glucose. Intrapartum hypoxia would have presented earlier with seizures and possibly renal failure. Pyridoxine dependency would result in intractable seizures. Septicaemia is unlikely with the virtually normal blood count. Intravenous sodium benzoate and sodium phenylbutyrate are commenced. She is transferred to the paediatric intensive care unit (PICU) for haemofiltration. Ammonia normalizes over 6 hours. While on PICU she suffers a seizure. Investigations: High plasma citrulline and absent urine orotic acid suggest citrullinaemia, subsequently confirmed with mutation analysis. She subsequently recovers and is discharged on a low protein diet, sodium benzoate and arginine.
577 CHAPTER TWENTY-NINE growth restricted, or who have mothers with maternal diabetes or who are seriously ill, e.g. sepsis or hypoxic– ischaemic encephalopathy. If severe or recurrent, or if it occurs in older infants and children, investigations should be undertaken to identify a serious underlying pathology (see Chapter 26, Diabetes and endocrinology). A hypoglycaemia screen should be performed at the time of low blood glucose (Table 29.8). It is essential to establish the presence or absence of ketones, as their absence is an abnormal physiological response. Figure 29.4 gives a diagnostic guide to metabolic causes. Prolonged fasts can be performed to provoke hypoglycaemia in order to permit investigations to be completed. However, this should only be done under strict supervision and once a fatty acid oxidation disorder has been excluded, as a prolonged fast could result in metabolic decompensation. at times of fasting or increased glucose demand, e.g. aerobic exercise. Clinical Hypoglycaemia is a common finding in children and is seen in many clinical scenarios. It is common in the immediate newborn period (see Chapter 11, Neonatal medicine), particularly in infants who are preterm or Fig. 29.3 Overview of glucose and glycogen metabolism. Glycogenesis Glycogen Glycogenolysis Glucose-1-phosphate Glycolysis Glucose 6-phosphate Fructose- 1, 6- bisphosphate Pyruvate Lactate Glucose GSD 0, IV GSD III, V, VI GSDI GSD VII GSD X, XII, XIII GSD XI Table 29.8 Tests comprising a hypoglycaemia screen Bloods Urine Blood glucose – confirm severity Blood gas Free fatty acids 3-hydroxybutyrate Insulin and C-peptide Cortisol and growth hormone Lactate Ammonia Acylcarnitines Plasma amino acids Urea and electrolytes, liver function tests Organic acids Ketones Reducing substances Fig. 29.4 Diagnostic guide to hypoglycaemia. GSD, glycogen storage disease 0, I, III, IV, IX; FAOD, fatty acid oxidation disorder; FBP, fructose-1,6-bisphosphatase deficiency. Hypoglycaemia Hepatomegaly ↑ lactate At fast Post prandial Yes No GSD I, FBP, FAOD GSD III, VI, IX Ketotic hypoglycaemia, GSD 0, galactosaemia, ketolytic defects FAOD, ketogenesis defects, hyperinsulinism No hepatomegaly Ketosis
29 578Metabolic medicine Glycogen storage disorder V (GSD-V) Glycogen biochemistry (see Fig. 29.3) Glycogen is a macromolecule found in the liver and muscles. It is the primary energy source between meals. It is composed of up to 60,000 glucose molecules joined by α-1,4 linkages with branching points formed by α-1,6 linkages at intervals of 4–10 glucose residues. Glycogenesis Glycogenesis occurs when excess glucose is available. The key enzyme is glycogen synthase. This exists in a dephosphorylated active (synthase a) and a phosphorylated inactive (synthase b) form. Glycogenolysis Glycogenolysis releases glucose from glycogen by two steps: • Phosphorylase splits the α-1,4 linkage, releasing glucose-1-phosphate. Like glycogen synthase, it has active and inactive forms. Glucose is the major inhibitor of its active form. • Debranching enzyme splits the α-1,6 bond, producing free glucose. Clinical The glycogen storage disorders (GSDs) are a diverse group and can be divided into hepatic, muscular and cardiac sub-groups (Table 29.9). Figure 29.3 broadly outlines their position within the pathway. The clinical phenotype is dependent upon the site of abnormal glycogen metabolism. Glycogen storage disorder I (GSD-I) Type I exemplifies the hepatic form. It is due to deficiency of glucose-6-phosphatase and leads to severe hypoglycaemia because of the inability to mobilize glucose from glycogen or to utilize glucose from gluconeogenesis. The children have hepatomegaly due to glycogen storage and a characteristic cherubic facies. Management is with regular feeds during the day and a continuous overnight feed to maintain normoglycaemia (see Principles of dietary treatment, below). Table 29.9 Classification of the glycogen storage disorders (GSDs) Presentation GSD Hepatic Ia, Ib, III, IV, VI, IX, 0 Muscle V, VII, X, XI, XII, XIII Cardiac II, III Question 29.3 Glycogen storage disorder type V (McArdle’s disease) A fifteen-year-old girl presents with symptoms of muscle pain and cramps and fatigue during brief, intense exercise. She has also noted episodes of dark-coloured urine accompanying these episodes. She is often able to resume exercise following a brief rest. Glycogen storage disease type V (McArdle’s disease), the most common muscle glycogenosis, is suspected. Which of the following statements about the disease are true (T) and which are false (F)? A. Hepatomegaly is a clinical feature. B. Her symptoms during vigorous exercise are due to the lack of pyruvate available for production of acetyl-CoA for the Krebs cycle. C. It is caused by a deficiency of the myophosphorylase enzyme, which leads to reduced glycolysis in muscle fibres and reduced production of pyruvate. D. She can resume exercise after rest because of the onset of free fatty acid oxidation within the muscle. E. The dark-coloured urine is from bilirubin. Answer 29.3 A. False. This form of glycogen storage disease affects muscles rather than the liver. B. True. C. True. D. True. The Krebs cycle is dependent on free fatty acid oxidation for production of acetylCoA. This process takes longer than pyruvate formation, which explains the ‘second wind’ phenomenon. E. False. Myoglobin (an oxygen and iron binding protein found in muscle tissue) is released into the blood due to muscle damage during intensive exercise and is responsible for her dark-coloured urine. The Krebs cycle and mitochondrial respiratory chain The Krebs cycle, also known as the tricarboxylic acid cycle (Fig. 29.5), is found in all cells except red blood cells, which lack mitochondria. It links the pathways of intermediary metabolism with the mitochondrial